prep test 6

87
KAPLAN LSAT PREP LSAT RELEASED TEST VI EXPLAINED A Guide to the October, 1992 LSAT

Upload: nipunscribd

Post on 15-Nov-2014

145 views

Category:

Documents


5 download

TRANSCRIPT

Page 1: Prep Test 6

KAPLAN LSAT PREP

LSAT

RELEASED TEST VIEXPLAINED

A Guide to the October, 1992 LSAT

Page 2: Prep Test 6

©1999 KAPLAN EDUCATIONAL CENTERSAll rights reserved. No part of this book may be reproduced in any form, by photostat, microfilm, xerography or

any other means, or incorporated into any information retrieval system, electronic or mechanical, without thewritten permission of Kaplan Educational Centers.

Page 3: Prep Test 6

© K A P L A N 1

SECTION I:

READING COMPREHENSION

Page 4: Prep Test 6

LSAT PREP ________________________________________________________________ LSAT Test VI Explained: Section I

2 © K A P L A N

Passage 1—Taft-Hartley Act(Q. 1-6)

Topic and Scope: “Right-to-work” legislation; specifically, the impact of such laws on thewages of minority workers.

Purpose and Main Idea: The author’s purpose is to examine a variety of studies, many ofthem in disagreement, about how (and indeed whether) right-to-work laws have impactedwages. This is done more in the manner of a survey than an argument; hence, the “mainidea” is little more than the general statement in lines 30-32 that studies like Carroll’s (andthe “pioneering” one of Ashenfelter) have “important implications,” most notably thatBlack workers’ wages may be depressed by right-to-work laws.

Paragraph Structure: ¶1 begins by describing what “right-to-work” is: a reaction to union-shop agreements that force everyone to become a member. In a state with right-to-worklaws, a worker need not join a union. Lines 9 and on switch to the impact of such laws, andthe view that that impact is negligible “has not gone uncriticized,” notably by Carroll. Heasserts (lines 20-25) that right-to-work hurts workers not directly but indirectly, byweakening union bargaining power and hence (lines 28-29) lowering wages.

¶2 shifts the focus to demographics, specifically to minority workers, whom the authorbelieves (lines 32-36) may be hurt more in right-to-work states than in states having a unionshop, since unions tend to have a positive impact on minority wages. Lines 41-66 are wherethe passage gets most torturous, and where you the reader have to be careful to get the gistof what’s going on and not get discouraged if the specifics seem to elude you. A goodstrategy may be to start with the “pioneering” Ashenfelter—i.e. someone the author likes;get a sense of his view, and then figure out what Ashenfelter is reacting against. Here thescope narrows even further, to compare two types of unions. Craft unions, Ashenfelterfinds (lines 49-53), have tended to exclude Blacks, and thus White craftspeople tend tomake more than Black craftspeople; but things are very different in industrial unions, where(lines 55-58) Blacks’ and Whites’ wages tend to be much closer.

All of this should make clearer what’s going on in lines 41-47. Instead of comparing unionvs. nonunion wages, Ashenfelter (and our author) imply, we are wiser to compare theimpact of right-to-work on different types of unions. If we do so, the passage implies, we willsee that the author’s indirect and tentative confirmation to the scenario posed in lines 32-36is Yes, Black unionized workers are likely to do better, and Black workers in right-to-workstates are generally going to be hurt economically—unless, as we see at the very end, theeconomy in general improves, thus creating a general demand for labor.

Page 5: Prep Test 6

LSAT PREP ________________________________________________________________ LSAT Test VI Explained: Section I

© K A P L A N 3

The Big Picture:

• Section Management strategies should always be in the forefront of your mind. Howdid many test-takers decide that this was a good passage to hold for later? It’s long.It’s boring (no offense to labor law buffs). It begins in a heavily factual way; theauthor’s views don’t appear early. Most of the passage is devoted to a factualrecounting of the views of others; the author is only occasionally present (e.g. lines11-12, and 20-32), and the author’s main idea is ambiguous, at best (notice that thequestions don’t include a typical “main idea / primary purpose” question.) Finally,new concepts are introduced right up until the final sentence; economic growth,labor shortages—it just never seems to end. . .

For these reasons, the reading challenge in this one is tougher for examinees than,say, Passages 2 and 3 in this section. Most people are better off saving a toughpassage like #1 until later in the Reading Comp. section when, having gotten pointsunder your belt and (one hopes) set aside time to spend, you can dig into a toughpassage in a relaxed and thoughtful way.

• You must begin every passage, especially a thorny one like this, with a solidfoundation, which means the opening 1/3 of text. Take your time! Read and thinkabout and paraphrase the opening sentences carefully. (In this passage, for instance,it was critical that you understood what “right-to-work” is, and what the pro and conissues are, before proceeding further.) If you go for a mere “skim,” you’re likely toneed one or more time-consuming re-readings later on, in order to get a handle onwhat’s what. Spending time thinking about the opening 1/3, in fact, saves time.

• Don’t lose sight of the passage’s scope as it moves—or in this case, crawls—along.Especially note places in which the author explicitly narrows the scope, such as here,when he moves from right-to-work’s impact on workers in general in ¶1 to its impacton minority workers in ¶2.

The Questions:

1. (E)As if it weren’t bad enough that the section begins with a tough passage, the passage beginswith one of the toughest question types, akin to Parallel Reasoning in the LR sections. Ontop of that, to get the answer, you have to reread a lot of context: Though the questionstem’s “literature” is mentioned at line 9, it’s not described until lines 13-17, by the critic ofthat literature, Mr. Carroll. He says that the researchers believe the right-to-work laws haveminimal impact because they have simply looked at whether the laws have decreasedunion membership and said, “Hey, they haven’t, so there’s no impact.” In the same way,(E)’s transit system asserts that a fare hike has had no negative effects simply becauseridership hasn’t decreased. In both cases, a look at sheer numbers prompts a hastyconclusion that may not be supported by other relevant data.

(A)’s conclusion weighs benefits and disadvantages and states one side’s argument for whya law should be passed. That’s a recommendation, not at all similar to the value judgmentregarding the impact of right-to-work laws found in lines 9-17.

Page 6: Prep Test 6

LSAT PREP ________________________________________________________________ LSAT Test VI Explained: Section I

4 © K A P L A N

(B) includes evidence that amounts to a value judgment (“The well-off can afford the tax”)but, similar to (A), concludes with a strongly implied recommendation suggested by theproponents: The tax should be passed. It doesn’t conclude that “X has had no impact”—which is what we need.

(C) The students pre- and post-curriculum change might seem to parallel the workers pre-and post- right-to-work laws. But nothing in lines 9-17 has a parallel to (C)’s charge ofunfair treatment. (C) would work better if it went like so: “Since there are as many studentssince the curriculum change as there were before, the change has had no effect on thestudent body.”

(D), far from concluding that a phenomenon (the new policy) has had no effect, concludesthat its effect has been major.

• Never succumb to the assumption that you need to, or should, attack LSATquestions in the order in which they’re printed. Feel free to skip tough ones insearch of the quick and easy points that you know are there. Somewhere.

2. (C)“This point of view has not gone uncriticized. Thomas M. Carroll...” These sentences linkCarroll to the controversial point of view in lines 9-11. Carroll takes exception, as (C) says,to the idea that right-to-work laws haven’t had an impact on wages. The rest of the ¶ goeson to elaborate how Carroll believes that the laws weaken unions and encouragemanufacturer/supplier collusion.

(A) Au contraire, this is the viewpoint Carroll rebuts.

(B) Carroll’s study does see right-to-work laws as an impediment to unions, but we’reexplicitly told (lines 21-23) that such laws do not reduce union membership.

(D) Carroll’s study does not support, but rather criticizes, earlier research.

(E) Carroll acknowledges that there’s an opportunity for collusion, but that’s all; we neverhear about the mechanics of doing so.

• Never answer a question based on your memory of the text. Go back to re-read thekey passage(s) first. Taking those extra seconds will pay off in more right answersand a net gain of time.

3. (A)Craft unions are mentioned once and once only: as part of the “important fact,” cited byAshenfelter, that minorities have been traditionally excluded from membership in suchunions and that therefore craft unionism increases the gap between Black and White wages.(A) paraphrases this idea that, through craft unions, more money is earned by members(largely White) than non-members (most Blacks).

Page 7: Prep Test 6

LSAT PREP ________________________________________________________________ LSAT Test VI Explained: Section I

© K A P L A N 5

(B) Nothing is said about increased membership in craft unions. If anything, the exclusionof a large number of workers would tend to depress the total.

(C) is a distortion of the comparison between craft and industrial unions. Yes, the latterhave more minority members and tend to reduce the Black/White wage differential, butwe never learn anything about the rate at which wages rise in different unions.

(D) We are told that right-to-work laws have a negative effect on wages of industrialworkers, but we can infer that they would also affect the wages of craft union members who,as Ashenfelter reports, are mostly white.

(E) misinterprets the point made at the end of ¶2 (which is rather far removed from thecraft union reference). We know that a healthy economy can create a demand for workersand raise their wages. But there’s no way to tell from the info given which group—craft orindustrial—would be more likely to experience wage increases under these circumstances.For all we know, the increase in wages brought on by labor shortages applies only to theindustrial sector.

• When the basis of a question is highly localized (as it is here, to lines 49-54),recognize that the further you move away from that detail, the more likely you are tomove away from the correct answer. (Example: (E) here, almost 15 lines removed fromthe craft union reference.)

4. (E)This one follows up on Q. 3, because right after the author polishes off craft unions (line54), he cites the more favorable situation in industrial unions, where Black workers havemade wage gains. Correct choice (E) is, in fact, lines 55-58 almost verbatim. It’sAshenfelter’s estimate, yes, but cited approvingly as if as fact by our author.

(A), (B), (C) All of the “Prior to 1947”’s relate to the situation prior to the Taft-Hartley Act.But that appears in ¶1, long before industrial unions take center stage in this passage!Industrial unions are never explicitly related to the pre-right-to-work era, so none of thesestatements is remotely inferable.

(D) Au contraire! (D) directly contradicts lines 55-58 as well as correct choice (E).

• Sometimes, more than one answer choice is wrong for the same reason.

5. (B)The question assumes a “Yes” answer as to whether right-to-work weakens unions, andgoes on to speculate about a counteracting force. The only place that that comes up is inthe last sentence, where we learn that a possible exception would be a right-to-work statewhere the economy is booming. In such a case a demand for labor might drive wages up.

(A) makes no sense. Decreasing the number of union shop agreements would most likelydecrease the economic power of unions even more.

Page 8: Prep Test 6

LSAT PREP ________________________________________________________________ LSAT Test VI Explained: Section I

6 © K A P L A N

(C), (E) Craft unions come up in the discussion of race-related wage differentials, but wecannot infer that either a decrease in craft union membership (C) or their decline in thelabor market (E) would counteract the decrease of unions’ overall economic power in right-to-work states.

(D) The idea of merging unions, whether weak or strong ones, is never mentioned, so wehave no idea whether a merger would make any difference.

• Think about where a question’s source in the passage is likely to be. Re-read thatsource, and pre-phrase an answer, before proceeding to the choices.

6. (D)This one should be manageable if you recognize the ¶ structure as we described it above.Carroll and Ashenfelter, each in his own sphere, take on misguided thinking about right-to-work laws, and the author merely gives them a forum to do so. So (D)’s “review ofresearch that challenges earlier research” is the closest to the mark.

(A) loses sight of the key question at the heart of the passage: Have right-to-work laws hurtworkers? (A) ignores that question, and makes it sound as if the only issue is how toconduct research.

(B) There are two pairs of competing ideas that take center stage here: the view that right-to-work has had no impact, vs. Carroll; and the tactic of comparing union workers to non-union ones, vs. Ashenfelter. And to “reconcile” means to bring together, whereas ourauthor seems to, overall, favor the two researchers mentioned.

(C) The passage could be said to “critique” right-to-work laws in the sense that such lawsdepress wages, and disproportionately so for minorities. So (C) could be considered half-right, but its latter half is dead wrong. The author shows no interest in exploring orinfluencing future change.

(E) No one “specific case” is cited. Yes, Ashenfelter is credited with a specific comparisonbetween craft and industrial unions, but that’s buried in lines 47-58 and doesn’t deserveglobal prominence. Furthermore, far from confirming earlier studies, Carroll rebuts theinterpretation of the previous “literature” of right-to-work laws, as it’s so called.

• When answer choices (like these) are highly abstract, be sure to pre-phrase ananswer first, and then go on to treat the language in the choices concretely—otherwise they’ll all sound good.

Page 9: Prep Test 6

LSAT PREP ________________________________________________________________ LSAT Test VI Explained: Section I

© K A P L A N 7

Passage 2—Women Physicians In China(Q. 7-12)

Topic and Scope: Women doctors in China; specifically, the need for women missionarydoctors in the late 1800s.

Purpose and Main Idea: The author’s purpose is to explain how it was that the influx ofWestern women doctors created opportunities for both Western and Chinese women; andthe Main Idea is pretty much the passage’s first sentence. The structure and gist of thepassage are made very clear from the very beginning—good news for test-takers.

Paragraph Structure: ¶1 focuses on the missionary movement, and the changes that led tosending women doctors to China. The most notable change was the growth, from 1870 on,of women’s mission societies, which were much more eager to send single women overseasthan were the traditional male-dominated mission boards; but the latter fell into line (lines25-30) over time.

¶2 describes the conditions that Western women doctors found in China—conditions thatyou might have mentally summed up as “initial unease, then widespread acceptance.”Then, as promised in lines 3-4, ¶3 explains the impact of these developments on Chinesewomen, who first made use of the women doctors’ services and then began to be medicallytrained themselves, thus becoming able to crack the medical “glass ceiling” of its day andearn an independent living. ¶4 sums up, in rather general terms, the progress for womengenerally that was signaled by all of the above-described developments.

The Big Picture:

• Start reading and thinking at line one! A passage like this one dramaticallyillustrates how much of the “game plan” an author can give away as early as the veryfirst sentence. Don’t wait for 12 lines to go by before you start thinking andparaphrasing. Get busy early!

• In Reading Comp., promises made are promises kept. The first sentence promises atreatment of how opportunities arose for Western as well as Chinese women; weshould expect each of those to be treated in turn. (And they are, in separate ¶s.)When an author makes promises, see to it that s/he keeps them.

• The concept of “gist” comes into play in ¶1. You don’t have to understand all of thedetails, chapter and verse, of how women doctors came to China. Get a rough senseof it—“it was women who sent the women overseas”—and wait for the questions todemand that you read and understand more deeply than that.

Page 10: Prep Test 6

LSAT PREP ________________________________________________________________ LSAT Test VI Explained: Section I

8 © K A P L A N

The Questions:

7. (E)This one is hard to pre-phrase, because the author says so very many things about Westernwomen working abroad. But since the question speaks about missionaries only and leavesdoctors out, you might have guessed that the answer must come out of ¶1, the ¶ that hasthe most to do with the missionary movement. Note, too, that all five choices say “beforethe 1870s,” which is ¶1’s time frame. For these reasons, you might have gone back toconsult ¶1 in more depth before attacking the choices. As it happens, lines 15-16 spell out(E)’s inference that the typical pre-1870 women missionary went overseas with herhusband.

(A) There were few single women missionaries pre-1870, but we know nothing about theoverall total.

(B) Au contraire, the author states that the women’s societies didn’t start financingmissionaries until 1870.

(C), (D) Both topics—women nurses (C) and the pre-1870 location of women missionarywork (D)—go unmentioned in the text.

• Attack each question stem carefully for clues. Note, for instance, that here in Q. 7 theabsence of a reference to doctors helps us determine which part of the passage toconsult for the answer.

8. (D)“Exclusively male.” That hearkens back to ¶1. Scan for the phrase: it’s present at line 22.Study the context: The uniformly male members were “uniformly uneasy about” sendingsingle women overseas. That “account[s] for [their] attitude,” as (D) has it, and ties into thepurpose of the detail overall, to show how things changed when women’s societies beganmaking the decisions.

(A), (B), (C), and (E) all fall short because they make no mention whatsoever of the genderissue which is, after all, what the question is based upon. (A) trumps up a religious vs.secular distinction, (E) a distinction between work abroad and at home; (B) evokes Chinesewomen, who don’t show up until ¶3; and (C) brings in “professional qualifications,” whichare never mentioned.

• Quite often a Reading Comp. question stem will mention a potent word or phrasefrom the passage without putting it in quotes. (Often it’s a proper name; sometimesit’s a phrase like “exclusively male” here; see also Q. 11, below.) Follow up all clues!Use the potent word or phrase to help you locate where the answer is to be found.

Page 11: Prep Test 6

LSAT PREP ________________________________________________________________ LSAT Test VI Explained: Section I

© K A P L A N 9

9. (A)Choices written this abstractly can be maddening. They all sound good at first reading,don’t they? Avoid a quagmire by pre-phrasing the organization on your own; it might golike so: “¶1 explains how single women doctors began to go over to China; and ¶s 2 and 3show the impact of their presence within China.” The passage’s wonderful first sentence isthe “situation” that correct choice (A) describes, and the rest of (A) is what we justdiscussed: the causes for that “situation,” and the results.

(B) The author never “refutes” the assertion she makes in the first sentence. (B) makes itsound as if the passage is a give-and-take debate between two sides, rather than an alreadyfully-reasoned argument.

(C) Even if there were an “obstacle” kicking off the passage—which there isn’t—(C)’s use offuture tense (“possible ways to overcome”) is a deal-breaker. This is an historical overview.

(D) “Predicament” is no more appropriate here than (C)’s “obstacle.” And this overview ofdevelopments in the latter half of the 19th century hardly ends with “a recommendation.”

(E) No drawbacks of having women doctors in China are mentioned. And what “eventualoutcome” could be “predicted”? All those people are dead now.

• By pre-phrasing an answer—however awkwardly or tentatively—to a question inwhich the choices are all abstract, you are less likely to be taken in by off-the-wallchoices, and more likely to locate the right answer quickly.

10. (A)According to the author, the reason the 1870s began the increase in overseas single womenmissionaries was the rise of women’s groups as sponsors, in contrast to the all-male boardsthat were doing the sending previously. The right answer should cut the cord between thewomen’s groups and the missionaries, and that’s what (A) does. That the majority ofoverseas single women missionaries got no sponsorship from women’s groups strikes atthe heart of the thesis in ¶1, and makes it necessary to search for another explanation for thesame phenomenon.

(B) speaks, perhaps, to the personal motivation of women wanting to serve overseas, butthat doesn’t affect the author’s analysis of why in the 1870s more single women began to doso.

(C) So women doctors were outnumbered by women teachers and translators. So what?Why else, except for the reason given by the author, did their number increase after 1870?(C) has no reply.

(D) and (E) don’t, either, concentrating as they do on the nature of the women missionarieswho went abroad. Nothing in these choices undermines the author’s contention thatfinancial support from the newly-created women’s societies was responsible for theincrease noted in the stem.

Page 12: Prep Test 6

LSAT PREP ________________________________________________________________ LSAT Test VI Explained: Section I

10 © K A P L A N

• Undermining an argument in Reading Comp. requires the same work as in LogicalReasoning: locating the evidence and conclusion, examining the link between them,and severing that link in some way. It’s a little harder here, since you have to dig theargument in question out of 60 or so lines of text. But the process is the same.

11. (D)The stem’s reference to “Western women . . . physicians . . . in Canton” clearly sends us to¶2. Rereading that ¶ for the reasons they became accepted reveals lines 33-38: Womendoctors were a great advantage because they could treat women patients and maintaintraditional female modesty. That last is the “cultural conventions” to which (D) refers,reinforced one ¶ later in lines 45-50.

(A) Only if the number of male doctors was low would (A) possibly help to explain theacceptance of female doctors. But (A) doesn’t tell us what that number was, so (A) fallsshort.

(B), (C) Talk about outside the scope! The Western sponsors and parishes are longforgotten by the time we get to ¶2. The local home parishes (C) have nothing to do withWestern women physicians’ reception in China; and the sponsors (B) are merely themechanism that got them there.

(E) And why would relations between Chinese officials and Western missionary boardsimpact at all on women doctors’ acceptance in China? Far-fetched . . . and unsupported bythe text in any case.

• The harder you have to work to justify an answer choice, the more likely it is thatthat choice is wrong. Use the text to make the selection process less difficult.

Page 13: Prep Test 6

LSAT PREP ________________________________________________________________ LSAT Test VI Explained: Section I

© K A P L A N 11

12. (D)Chinese medical practices come to the fore in ¶s 2 and 3. We don’t learn much, except forthe issues of female modesty mentioned during our work on Q. 11. ¶3 asserts that manyChinese women would have been unlikely to seek medical treatment but for the presenceof a female doctor in whose presence they’d (inferably) be more comfortable. This leads to(D)’s inference that, customarily, male doctors didn’t treat female patients.

(A) contradicts the passage: many Chinese women began to avail themselves of Westernmedicine at this time (lines 45-50).

(B) More women may have been treated at home than in hospitals, if the alternative was tosee a male doctor. But (B)’s conclusion about overall medical care is unsupported by thetext.

(C) is wildly contrived. No financial obligation on the part of the doctor to the family isever mentioned. All we learn is that medicine helped some Chinese women becomefinancially independent.

(E) If anything, ¶3 suggests that Chinese women didn’t ordinarily have the sameeducational opportunities that young men did, especially in medicine. (E) contradicts this.

• Be on the lookout for overlapping questions in Reading Comp.: two or morequestions that test, or revolve around, the same issues. Never assume that everyquestion is independent of the others. Here, both Qs. 11 and 12 are related to theChinese convention of female modesty mentioned in ¶s 2 and 3.

Page 14: Prep Test 6

LSAT PREP ________________________________________________________________ LSAT Test VI Explained: Section I

12 © K A P L A N

Passage 3—Early Music Movement(Q. 13-20)

Topic and Scope: Early music; specifically, the movement to have music performed as itwas performed when it was written.

Purpose and Main Idea: The author’s purpose is to explore the “questions” raised by thismovement (which she describes as resembling a “crusade”), and that the questions are“profound and troubling” (lines 12-13) conveys her main idea.

Paragraph Structure: ¶1 describes the early music movement, shoots it a little barb by wayof the “crusade” reference, grants that the movement has been of value, and then presentsthe main-idea sentence that the rest of the passage follows up on: the “profound andtroubling questions” raised by the movement.

One would expect that what follows would explore at least some of those questions, andsince two ¶s follow, it’s not surprising that those questions are two in number. To make thestructure even clearer, the author supplies the nice “continuation” Keyword phrase “inaddition” at the beginning of ¶3 to essentially say “that was one problem; now here’sanother.” ¶2 explains that demanding that a piece be played on instruments availableduring its composition carries with it a built-in problem: What if a piece was composedwith instruments in mind that hadn’t even been invented yet? In that case, performing thepiece today on the earlier instrument that was available to the composer at the time wouldseem to degrade the artist’s vision. Beethoven’s first piano concerto is cited at length as onesuch piece.

¶3 poses a different “troubling” issue, expressed generally in lines 34-36 and illustrated bythe tempo issue. The gist of it is that the conducting (lines 37-46) and setting (lines 46-60) oforiginal tempos were both determined by different historical conditions from ours; anddenying that amounts to “inadvertently [divorcing] music and its performance from . . .life”—something the author finds troubling and obviously opposes.

The Big Picture:

• Strive to become a good anticipatory reader. Consciously anticipate that when anauthor calls something “a crusade,” s/he may turn out to be critical of it. Consciouslyanticipate that when something “raises profound and troubling questions,” almostimmediately those questions (at least one main one, and possibly even more thanone) will be raised and addressed. Stay ahead of the author rather than several stepsbehind.

• The Beethoven example in ¶2, and the two tempo examples in ¶3, are pretty technicalfor non-musicians to understand. Content yourself with understanding them inbroad outline (as discussed above), and delve deeper only when the questions seemto demand that you do so.

Page 15: Prep Test 6

LSAT PREP ________________________________________________________________ LSAT Test VI Explained: Section I

© K A P L A N 13

The Questions:

13. (D)Line 30 appears in ¶2, whose purpose is to attack the assumption on the part of early musicadvocates that composers necessarily write for the instruments that exist at the time. Rereadthe whole deal, from line 20 on. A piano “exactly contemporary with” Beethoven’s firstpiano concerto would, we are told at the end, frustrate Beethoven. Sure, it would irritatehim to have to play the “bad” F-natural, because the instrument of the time lacked the highF-sharp that existed only in the composer’s imagination and not on the instrument itself. As(D) has it, a piano contemporary with Beethoven’s first piano concerto would be incapableof playing the high F-sharp that the melody calls for. (Note that the actual score was writtento accommodate the range of the limited piano—hence the “wrong” note, the high F-natural that must have driven Ludwig batty.)

(A) The phenomenon of the inaudible time-keeping piano comes out of ¶3, and is notassociated with either Beethoven or line 30.

(B) Au contraire—check out lines 23-25 again. Pianos in Beethoven’s day lacked the F-sharp,not the F-natural.

(C) Mozart and Haydn appear only in ¶3, so there’s no reason to connect them to ¶2’spiano, and no reason to assume that all three composers couldn’t have used the same pianotype.

(E) First of all, as far as we know, Beethoven only contemplated revising his earlier work.And besides, it wouldn’t have been the piano contemporary to the first concerto thatwould have prompted Beethoven’s revisions, but rather the more expansive and versatilepiano that came later on.

• When a question mentions a line reference, be sure to think about the overallpurpose of the ¶ in which the line reference appears. Often, context is everything.

• Beware of choices that spring from details from the wrong part of the passage. (A)and (C) are good examples of this common wrong answer type.

14. (A)This one is pretty much a slam dunk, reflecting as it does the crucial sentence in lines 11-13:The early music approach “raises profound and troubling questions” with respect toperformance. As mentioned above, this sentence is pregnant with promise—what are theseproblems?—and the answer is found in the remaining two ¶s.

(B) gives the early music movement too much credit—“largely successful” is not ajudgment the author makes—and to narrow the rest of the passage discussion merely to“the use of obsolete instruments” leaves out most of what drives ¶s 2 and 3: Beethoven’simagined piano, the time-beating problem, and the tempo issues.

(C) keys off of a problem alluded to in ¶2 but falls far short of summing up the entirepassage.

Page 16: Prep Test 6

LSAT PREP ________________________________________________________________ LSAT Test VI Explained: Section I

14 © K A P L A N

(D), (E) Neither the lack (D), nor the incomplete use (E), of information is ever alluded toas central to the author’s interest in the early music movement.

• When you’re confident about an answer choice early in the set, you should still atleast glance at the remaining choices, but in a different way: with less respect. Don’twaste time evaluating them in depth; your task has shifted to simply making surethey’re as bad as they need to be, considering that you’re pretty sure you alreadyhave the winner. It may not seem like this will save you much time, but a fewseconds gained on a handful of questions could very well add up to an extra minuteor two by the end of the section.

15. (D)The first piano concerto is introduced at line 20, and as such it acts as the “evidence,”mentioned in the previous sentence, that such eminent composers as Beethoven imaginedweird notes that the instruments of the day couldn’t play. In particular, the piece featuresthat high F-sharp that only some hypothetical future piano could hit. (D) expresses notonly the point of mentioning the concerto, but the essence of ¶2 overall as a problem withthe early music movement’s premises.

(A) has it backwards. Beethoven was forced to include a “wrong” note in his score becausethe pianos at the time couldn’t accommodate the correct melody. Only later did the rangeof pianos expand.

(B) Au contraire, Beethoven explicitly did anticipate the more sophisticated piano of thefuture as he imagined notes that contemporary pianos couldn’t hit.

(C) We’re told only that Beethoven thought about revising his earlier works, from which wecan’t infer that a revised version of his first piano concerto actually exists. Thus,Beethoven’s first piano concerto is not used here to show that early music advocates stickto original scores despite later revisions, because we don’t even know that there is a revisedscore for this piece.

(E) Au contraire again—the piano available at the time Beethoven wrote the first pianoconcerto frustrated the great composer’s intentions by lacking the high F-sharp called forby the melody.

• Check the context of any detail mentioned in a question stem. Often the detail’spurpose is made clear (as it is here) by the text that directly precedes or follows it.

• Beware of au-contraire choices, choices that provide the opposite of what we’relooking for. Here, there are no fewer than three wrong choices that seem to contradictthe information in the passage.

Page 17: Prep Test 6

LSAT PREP ________________________________________________________________ LSAT Test VI Explained: Section I

© K A P L A N 15

16. (B)Any reference to tempo must direct your attention to ¶3, and the reference to theMozart/Beethoven movements to lines 46-56. The last movement, we’re told (lines 49-50),was played slower in the 19th century than it is now, and the author explains why: Today,audiences don’t applaud after each movement (as they used to) but only at the very end,and as a result “musicians [are] forced into extra brilliance at the finale in order to generateapplause.” If that habit were broken, if concerts went back to the old way—here’s (B)—thenit would be reasonable to suppose that the musicians wouldn’t need to crank the tempo upat the end to generate audience enthusiasm; they’d probably slow it down to the oldtempo.

(A) refers back to the wrong ¶. It’s ¶2 that is concerned with instrument design.

(C) is the current practice, the one that causes a fast-paced finale.

(D) The inaudible piano is a separate issue from the one alluded to in the question; itdrops out as of line 46.

(E) Nowhere are we told that Wolfgang Amadeus and Ludwig Van did their ownconducting, not to mention what relation, if any, that would have to tempo.

• Even when a question stem doesn’t direct you to a specific ¶ or sentence, you need toconsider where in the passage the relevant text is likely to be found. This is why it’sso important to construct a mental roadmap of the passage as you read through thefirst time.

17. (B)The second “troubling question” raised by the early music movement is its “divorc[ing of]music and its performance from . . . life” (lines 34-36). That criticism is followed up in ¶3by two specific examples: the inappropriate inclusion of the “silent” time-keeping piano,and the change in tempos due to changes in audience behavior. (B) is right on the money.

(A) The author is not in the business of “undermining” his own assertion, and the onlythings he “rejects” are some of the premises of the early music movement, but that’s notwhat (A) refers to.

(C) ¶3 begins not with a statement of the movement’s assumptions, but an indictment ofone of its practices. That indictment is subsequently supported, not undermined.

(D) That phrase “frequently provided” is curious—what’s it based on? Anyway, ¶3’sevidence is approvingly cited, not “critically evaluated.”

(E) The “two specific cases” (the silent piano; the tempo examples) are preceded by acriticism (lines 34-36) that they’re there to support. (E) has it backwards.

• Use your Kaplan materials to get more familiar with the abstract rhetoric terms usedin LSAT questions like this one, especially “generalization,” “assumption,” and“evaluation.” (See also Q. 24, below.)

Page 18: Prep Test 6

LSAT PREP ________________________________________________________________ LSAT Test VI Explained: Section I

16 © K A P L A N

18. (D)You can’t weaken an argument until you’ve confirmed its evidence and conclusion. Thephrase “can readily be explained by” tells you that the conclusion has just been mentionedand the evidence is about to follow. So: The tempo differences between Mozart’s andBeethoven’s day and our own is explained in terms of audience custom: Since in the olddays audiences applauded throughout the piece, the tempos during the piece were fasterback then; since today’s audiences only applaud at the end, the tempo at the end is fasternow. But if, as (D) says, the applause during the piece was minimal—if the real applausecame at the end—then the customs of yesterday’s and today’s audiences aren’t all thatdifferent, and an explanation of the differences in tempo must be sought elsewhere.

(A) and (E) are outside the scope: Neither the musicians’ amount of training (A), nor theaudience’s knowledgeability (E), is remotely cited here, let alone cited as relevant to thetempo issue.

(B) The fact that breaks may have been longer then than now doesn’t relate in anymeaningful way to the explanation offered by the author, which focuses on the effect ofaudience reaction on tempo. What happens at the end of each section is the relevant issuehere. We cannot infer what effects on audience reaction, and by extension, performancetempo, the length of the breaks between sections would have.

(C) may be tempting, since the author does seem to see a relationship between the tempo atwhich music is played and the audience applause that is desired. But (C) assumes that thephrase “concern with the audience response” translates to the seeking of applause—whichit needn’t; and the argument certainly doesn’t hinge on there being parity of concernbetween yesterday’s musicians and today’s. So even if (C) is true, the author’s explanationis not affected.

• Be careful when reading the question stem; the inclusion of the word “inferred” heredoesn’t make this an Inference question. There’s a big difference between lookingfor the choice that must be true based on the text and the task we’re presented withhere—finding the choice that, if true, would weaken the argument.

19. (D)The important word in the stem is “recordings,” which directs us to ¶3 in general and lines37-46 in particular. You’ll recall that this takes us to the first of two extended examples ofwhat the author sees as a disturbing aspect of the early music movement, its tendency toseparate the performance of music from the real life of a bygone day. The recordings inquestion feature an obtrusive piano thump-thumping the time (lines 40-42); that piano isnominally “accurate,” because it did exist in the 1700s, but the fact that 18th-centuryaudiences barely heard it (lines 42-46) means that it is accurate in name only; the recordings’sound itself is quite different from the 18th-century sound. The sense is that in this respectthe movement is crossing itself up, or as (D) puts it, performing music most UNlike theway it sounded at the time of its composition.

(A), (E) Both can be rejected out of hand because of their references to issues introducedlater in ¶3: tempo and “intensity and excitement,” respectively.

Page 19: Prep Test 6

LSAT PREP ________________________________________________________________ LSAT Test VI Explained: Section I

© K A P L A N 17

(B) seems to tie lines 4-8 together with ¶3, resulting in an extremely weird statement. Wheredoes “betray the influence” come from, and where is the passage ever concerned withinfluences on the advocates of early music? Strange.

(C) has it exactly backwards. By insisting on a instrument that was present in the 1700s andending up with a sound that was not, the early music recordings are sacrificing “aestheticintegrity” for the sake of “historical authenticity.”

• The harder you have to work to justify an answer choice, the more likely it is to beincorrect. Study the context of the detail in question, but don’t wander too far fromit. (Many wrong answers do just that.)

20. (C)Again, as in Q. 19, remember that ¶3 explores the dichotomy between music and life. Inlife, says the final sentence, “our concepts of musical intensity and excitement have . . .changed,” which makes it ridiculous to insist on the original tempos of Mozart and Haydnsymphonies, since those tempos were based on audience custom; and customs, as (C)points out, change. We applaud differently because our tastes have changed. Only (C) evenremotely picks up on this.

(A) refers back to the time-keeping piano earlier in ¶3. We’re way past that detail by thetime audience applause is under discussion.

(B), (D) Both do speak to changes in audiences—their altered expectations of musicians’ability (B) and their greater appreciation of musical structure (D). But neither change is anissue mentioned or even alluded to in the passage; all the author focuses on is audienceapplause as a reflection of excitement and dash.

(E) If so, then audiences today would likely applaud after those exciting early movements.But they don’t. (E) is another au-contraire choice, contradicting the sense of the passage.

• Questions aren’t totally independent of each other. Use your work on one questionto help you with others that seem to hinge on the same issue or the same part of thepassage.

Page 20: Prep Test 6

LSAT PREP ________________________________________________________________ LSAT Test VI Explained: Section I

18 © K A P L A N

Passage 4—U.S. Steel Industry(Q. 21-27)

Topic and Scope: The U.S. steel industry; specifically, the characteristics and relativecondition of the three major types of steel producers.

Purpose and Main Idea: The author’s purpose is clearly announced in sentence 1: to showthat some branches of the steel industry are doing better than others in these troubledeconomic times. To achieve that purpose, he needs to describe those branches and thenexplain the success of some and the weakness of others. And that’s just what our authordoes, carefully crafting his explanation as to why the minimills and specialty-steel mills aredoing O.K. while the integrated producers are hurting.

Paragraph Structure: The first sentence of ¶1, as we’ve just noted, announces the topic,scope, and purpose about as explicitly as any test-taker could wish. The rest of the ¶ namesand describes the three branches of the industry. If you sensed that the branches’differences would be crucial to the passage and hence to the questions, you may havetaken a few seconds to make a quick list of what’s what, like so:

INTEGRATED: process iron ore & coal/wide range of steel/bigMINI: reprocess scrap steel/low-quality/limited range/cheap stuff/smallSPECIALTY: small/use scrap steel/expensive products/own R&D dept

¶2 begins by setting the mini and specialty mills apart from the larger integrated mills—they’ve avoided the economic problems of the latter and some are even “quite profitable.”The ¶ then focuses on the advantages of the two smaller branches, one of which theyshare—a use of new technology. Furthermore, each has its own separate advantage, we’retold: Minimills produce only a few products (we heard that in ¶1) to be sold close tohome, while specialty-steel mills are flexible to the customers’ needs. This informationallows us to expand on the lists above in an effort to continue keeping track of the variouscharacteristics of each type of mill.

Having heard about how well the two smaller branches are doing—and keeping theopening sentence in mind—we’re eager to hear about the woes of integrated producers,and that’s just what ¶3 presents. Lines 29-32 are a laundry list of woes, but it seems that thebig problem, outlined in lines 32-49, is an inefficiency inherent in the entire integratedproduction process, something that (lines 39-49) the U.S. companies share with otherintegrated producers around the world.

After ¶3 has explored the weaknesses within integrated producers, ¶4 goes on to explicitlycompare them to their smaller and more profitable brethren, and the discussion is detailedbut not out of control. Note, for instance, that ¶4 begins by picking up on ¶3’s technologytheme, highlighting the minimills’ and specialty mills’ technological efficiency. Note, too,that lines 57-63 simply repeat what we’ve already heard about minimills, that they sell anarrow range of cheap products close to home. That’s nothing new. Now it’s merely put tothe service of the first sentence of ¶1, which is reinforced at the very end: The big guys arehurting; the little guys are doing relatively OK.

Page 21: Prep Test 6

LSAT PREP ________________________________________________________________ LSAT Test VI Explained: Section I

© K A P L A N 19

The Big Picture:

• When you’ve determined the author’s purpose, be sure to keep it in mindthroughout. Here, it’s very easy to get lost in the morass of detail about steelproduction unless you keep remembering the initial point from sentence 1: theauthor’s desire to show that some branches of the steel industry are doing betterthan others. Having that in mind every step of the way keeps you focused rightthrough line 65.

• Many passages are based on some contrast or other. But some are so heavily basedon contrast that your main job becomes getting the contrasts under control. Here, wehave the basic differences among the three branches, as well as the morefundamental contrast between integrated producers (hurting) and the other two(doing well). Making lists of traits or terms associated with each mill type can helpyou manage the information much the way scratchwork helps in Logic Games.

The Questions:

21. (C)(C) reflects the author’s stated purpose, the results of his investigation into the state of theU.S. steel industry, and the structure and content of that investigation. Its wording mayseem detailed—but, then, so is the passage. And all of the wrong choices have profoundproblems:

(A) alludes to a comparison between the U.S. and other nations that only occupies lines 39-49, and even then it’s a comparison between integrated producers only. “Other nations” donot merit inclusion in a main idea discussion for this passage.

(B) asserts an unwarranted distinction and then proceeds as if minimills were the wholestory.

(D) highlights the 19th century—again, merely a detail late in ¶3—and implies an historicaloverview when the author’s clear focus is on U.S. steel today.

(E)’s misplaced focus on the future is as egregious as (D)’s focus on the past. (E) also errs byblurring the distinctions among the three branches of the industry.

• A passage’s main idea can be expressed in many ways. Look for the one choiceamong the five that sums up the author’s topic, scope, and purpose, but don’t insistthat your pre-phrasing match up exactly with what the testmakers give you.

Page 22: Prep Test 6

LSAT PREP ________________________________________________________________ LSAT Test VI Explained: Section I

20 © K A P L A N

22. (A)Since four of the choices are true of minimills, it is likely that the right answer will insteadbe true of one of the other branches. Two effective tactics here: (1) using the passage (oryour homemade lists) to locate minimill details, and throwing out choices that reflectthem, or (2) seeking out a choice clearly associated with another branch but not withminimills (or, of course, one that has nothing to do with any branch). A clear contrastcomes in lines 23-28, and that generates the correct answer: It’s the specialty mills that“preserve flexibility in their operations” (A), in contrast to the minimills’ “narrow range.”

(B), (E) Lines 23-28 and 59-61 indicate the minimills’ focus on local sales (B) and limitedproduct range (E).

(C) Minimills work only with scrap (lines 11-14), and have dispensed of the iron-smeltingprocess, “including the mining . . . of raw materials” (lines 52-56).

(D) Like specialty mills, minimills “take advantage of new steel-refining technology”(lines 21-22).

• Your two options in an EXCEPT question are to locate the “odd man out” directly, orreject the wrong answers in turn. Usually, a combination of the two techniques willyield the point.

23. (C)The reference to Japanese integrated producers at line 43 is preceded by “...but this cannotexplain why,” signaling that the purpose of the reference must be to rebut some assertionor explanation. Indeed, lines 39-42 mention a conclusion that “one might” make, that oldlabor-intensive machinery is the reason for the financial weakness of the U.S. steel industry,but that conclusion won’t fly. If it were so, how come the Japanese companies, whosemachinery is new and less labor-intensive, are also having financial woes? The Japanesesituation shows, according to the author, that the weakness of the U.S. industry must not bedue to the labor-intensive machinery. (C) explains that use to which the detail is put.

(A) Au contraire, the Japanese and U.S. integrated producers have economic woes incommon.

(B) Au contraire, lines 46-49 confirm that ALL integrated producers share the same“common technological denominator”: inefficiency.

(D) Hardly. See lines 46-49, “an inherently inefficient process” since the 19th century.

(E) may be a true statement, but not one that the Japanese counterexample sheds any lighton.

• When a question makes a line reference, be sure to consider its entire context, notjust its immediate context.

Page 23: Prep Test 6

LSAT PREP ________________________________________________________________ LSAT Test VI Explained: Section I

© K A P L A N 21

24. (E)As we’ve seen, ¶3 outlines the problems plaguing integrated producers, after which onepossible reason for poor performance (labor-intensive machinery) is rejected and another(inherent inefficiency) is proposed in its place. All of that is (E) in a nutshell.

(A) ignores lines 29-39’s detailing of the troubles of integrated producers, and then mixesup what follows: If anything, first we get a hypothesis (i.e. a working theory) that’scriticized, followed by an opposing view that’s supported.

(B) is just words. The author’s blaming the industry’s problems on inefficiency rather thanmachinery does not constitute the “resolution” of a “debate.”

(C) A dilemma is an inherent contradiction. ¶3 points to no such paradox.

(D) More words, ignoring lines 29-39 altogether. And there is no move from the specific tothe general in ¶3.

• Don’t just take words like “hypothesis” and “dilemma” at face value. Be sure youknow what they mean, and factor in their meaning when considering answer choicessuch as these. Otherwise virtually all the choices will seem tempting.

25. (E)Your lists of details can be exceptionally useful in questions like this one. (So can areminder that ¶s 1 and 4 are the main ¶s comparing the three branches of the steel industry;almost certainly one or both of those ¶s will yield Q. 25’s answer.) The fundamentaldifference between integrated producers on the one hand, and specialty-steel mills (dittominimills) on the other hand, is that the former smelt iron ore and coal while the latter justwork with scrap. Lines 50-57 spell out that both of the smaller branches “have dispensedalmost entirely with” such “front-end” elements as (E)’s blast furnaces.

(A), (D) Both are minimill details (lines 23-28). Specialty-steel mills retain flexibility.

(B) Au contraire, lines 17-20 tell us that specialty-steel mills share relative economic healthwith the minimills.

(C) distorts the example of the Japanese steel industry: The only Japanese counterpartsmentioned in the passage are the integrated producers, discussed in ¶3.

• If the answer to a question could come from two different ¶s—as in this case from ¶s1 and 4—be sure to check both thoroughly. Do NOT go just for the first detail orchoice that “looks good”!

Page 24: Prep Test 6

LSAT PREP ________________________________________________________________ LSAT Test VI Explained: Section I

22 © K A P L A N

26. (A)An interesting question of the “analogy” or “parallel reasoning” type. Four of the choicesface similar problems as integrated producers; we must find the odd man out. Your besttactic is to refresh your memory as to the problems of integrated producers, i.e. to reread¶3, and then skim the choices for parallels. One way or another you should be left with (A).It seems as if the only problem that integrated producers are not faced with is a shortage ofraw iron ore and coal. While everything else seems to be going wrong for the integratedmills, a lack of raw materials is never mentioned as a problem.

(B) shares the problem of “excessive labor” costs (line 31).

(C) describes the problem of heavy capital expenditures cited in line 31, and again in lines50-54.

(D) speaks to “manufacturing inflexibility” (line 32).

(E) Old and “less automated” equipment is the problem here, as it is for the integratedproducers in lines 32-34.

• Remember that picking a wrong choice simultaneously means that you’re rejectingthe right one. When reviewing your wrong choices, keep asking yourself why yourejected the credited choice. By doing so, you’ll be able to recognize bad habits thatyou can start to break.

27. (E)By this time, your familiarity with the problems facing integrated producers is probablykeen enough that you can study each choice, in whatever order, and discern fairly readilywhich one best matches up with the author’s explanation for the condition of this segmentof the steel industry:

(A) has to do with a nation’s economic health, not that of one of its industries.

(B) If integrated producers share a characteristic with the small and profitable specialty-steel mills, then that weakens any explanation about why the former are doing badly.

(C) The Japanese integrated producers are also in trouble. If the ones in the U.S. mimickedtheir quality, reduced energy, and reduced labor, then matters might improve, but that’snot what we’re looking for here. The fact that they’re adopting the Japanese style doesn’t tiein with why they are currently in the condition the author describes.

(D) speaks to efforts on the part of integrated producers to improve their lot; again, as with(C), that is outside the scope of the question of why they are down on their luck in the firstplace.

Page 25: Prep Test 6

LSAT PREP ________________________________________________________________ LSAT Test VI Explained: Section I

© K A P L A N 23

(E) suggests that the “front-end” task of iron-smelting—in which we know integratedproducers are engaged—is capital-intensive and unprofitable (lines 50-57), so much so thatin other countries it requires government subsidy. This is solid evidence that, as the authoralleges, iron smelting is indeed a drag on the integrated producers’ economicperformance.

• Strengthen/Weaken the Argument questions are more common in the LR sectionsthan here in Reading Comp., but as we see from this question and Qs. 10 and 18earlier, they do surface occasionally. Our task, just as in LR, is to consider eachchoice as true, and evaluate the effect it would have on the argument at hand. Also,we would expect such questions to contain the classic wrong answer types: ones thatsomehow fall outside the scope of the passage or the ¶ under consideration, or onesthat do the opposite of what the question is looking for (like weakener (B) here).

• Keep what you’re looking for firmly in mind. When looking for the choice thatsupports an argument of why things are bad, avoid choices (like (C) and (D) here)that posit what is being done because things are bad. Those are two completelydifferent issues.

Page 26: Prep Test 6

24 © K A P L A N

SECTION II:

LOGICAL REASONING

Page 27: Prep Test 6

LSAT PREP ________________________________________________________________ LSAT Test VI Explained: Section II

© K A P L A N 25

1. (C)“Law without enforcement is not law.” So to the author, a necessary condition for real lawmust be “enforcement.” That’s not one of the answer choices per se, but the author goes onto define “enforcement” not just as announcing forbidden acts, but as punishing those acts,and “without favor.” This is what (C) is getting at: the author’s deeper definition ofenforcement—that without which “real law” doesn’t exist.

(A) lacks context. Our author would consider impartial and just use of power as a part ofreal law, but not its “arbitrary and unjust” use. So (A) leaves matters too vague.

(B) Law that merely authorizes enforcement, without its being carried through, would fallshort of the author’s definition of real law.

(D) distorts the stimulus. The author sets out to specify when law is and is not “real,” andthat’s independent of whether someone “understands law’s purpose.”

(E) Sentence three makes it clear that real law “is not merely” (E)’s definition, but involvesthe punishment of violators as well.

• It is often difficult to pre-phrase answers to Inference questions, unless the questionexplicitly asks for the author’s main point or conclusion. In that case a solid attemptat pre-phrasing can really pay off.

2. (A)“[T]he accumulating data”—the list of ailments suffered by runners, both newcomers andveterans—are the evidence that “suggest” the conclusion: The human body isn’t built towithstand jogging’s stresses. Well, the first sentence merely indicates a correlation that hasbeen seen between jogging and the ailments. In other words, the fact that these ailments are“seemingly connected” to jogging tells us that joggers have these ailments, but not thatjogging necessarily causes these ailments. However, to draw her conclusion, the authormust believe that the connection is in fact causal, choice (A). If she didn’t so believe, shewouldn’t use this evidence to draw her categorical conclusion that the data suggest that thehuman frame is not built to withstand the rigors of jogging.

(B), (D) Both bring up an unwarranted distinction between jogging and “other sports.”Whether these same ailments are or are not present in other sports is beside the point in anargument about the dangers of jogging only.

(C) goes against the grain here—the author explicitly rules out differences between joggers’experience levels as a factor in injuries.

(E) “The human species” is way too broad—the scope here is limited to joggers.

• Use Kaplan’s Denial Test to confirm whether you’ve identified an author’s necessaryassumption. Remember how it works: if the answer is a correct assumption, itsdenial will contradict the text.

Page 28: Prep Test 6

LSAT PREP ________________________________________________________________ LSAT Test VI Explained: Section II

26 © K A P L A N

• Make sure you’re clear on the difference between causation and correlation.Sometimes, as is the case here, an author will treat evidence that two things appeartogether (that is, a correlation) as evidence that one causes the other. Evidence of atrue causal relationship must be very explicit, and here, we really have no hardevidence that jogging causes the ailments listed.

3. (D)Though this is nominally an Inference question, the stimulus has the sound of a paradox:There’s no conclusion per se, just two facts connected by a “however.” First we learn howthe Pitcombe students label themselves politically (and notice that the percents—25%conservative, 24% liberal, 51% middle of road—add up to 100%, so everyone’s covered).Yet a whopping 77% of the group hold a liberal position on a certain set of issues. Well,we’d expect self-proclaimed liberals to do so, but even if every single Pitcombe liberal plusevery single middle-of-the-roader held the liberal position, that would only account for75% of the students; the remaining 2% would have to come out of the conservative wing.So (D) has to be true: At least some conservatives must hold that liberal position.Otherwise the 77% figure could not be achieved.

(A) As we’ve just seen, IF all self-described liberals and middles hold the liberal position,then some conservatives must. And if some of the liberals and middles do NOT hold thatposition, then that just means that more conservatives must hold it. So (A) is by no meansnecessarily true; the math doesn’t require it.

(B), (C) It’s not clear whether “not endorsing” a position is the same as opposing it, buteither way, (B) and (C) cannot be inferred. If you assumed that “not endorsing” meansopposing, then you may have concluded that (B) is possible only, while (C) is impossible.

(E) Just as “opposition” is technically never mentioned, so is “a conservative position”never mentioned. And after all, even if some liberals do not hold the liberal view, it needn’tbe true that some of them support a conservative stance.

• When an argument throws numbers—especially percents—at you, do some quickarithmetic and see how (and whether) all the data jibe. Often the question will hingeon an overlap between groups, either one the author has failed to see or, as here in Q.3, one that has to exist.

4. (E)Perusing the stem first confirms that while Victor is trying to rebut Lenore, he does sobadly—and we have to identify the weakness. Lenore’s point is the virtual impossibility ofwriting history objectively, her evidence the inevitable personal bias of historians. Aproper rebuttal would attack that “inevitability,” or mention some unbiased historians byname, or point out other factors that might minimize bias. Instead, Victor simply says“Hey, some bias and biased people have been detected; therefore, those ‘detectives’ at leastmust be objective.” Hello? Cannot a biased individual perceive someone else’s bias? Surehe can: Such a person might be skilled at detecting the biases in others yet not be aware ofhis own. Identifying someone else’s bias is not, then, proof of one’s objectivity, and Victor’srebuttal fails because of (E).

Page 29: Prep Test 6

LSAT PREP ________________________________________________________________ LSAT Test VI Explained: Section II

© K A P L A N 27

(A) Two problems. First, Lenore is not attacking bias, as (A) would have it; she is arguingfor its inevitability. Second, yes, Victor’s logic is faulty, but he isn’t necessarily biasedhimself.

(B) Since Victor isn’t arguing that ALL historians are unbiased, he needn’t address anyexceptions to that generalization. What he needs to do is prove that some historians are notbiased; and as we’ve seen, he falls short in that effort.

(C) To rebut Lenore he doesn’t have to provide chapter-and-verse documentation aboutthe bias that’s been found. He must confirm the objectivity of the “detectives.”

(D) Victor’s argument doesn’t rely on rejecting a catalog of biases, just on proving whetherthe detectors of bias are themselves bias-free. Which, of course, it fails to do.

• When you’re asked to identify an argument’s flaw, sometimes thinking about whatmight make the argument sound can lead you to the flaw at hand. Here in Q. 4, forinstance, considering the kind of effective rebuttal that Victor could construct mayhelp you see where his effort fails.

5. (B)Why must the thieves have entered through the basement (i.e. below ground level)?Because the security guard says they didn’t come in on ground level or above. That isdifferent from evidence proving that alternative means of entrance were not used. Theessence of the logic is a flatly factual statement of a phenomenon that must have occurred,but a statement that is based on a subjective assessment that dismisses other alternatives.That describes (B) pretty well, as well as the stimulus, doesn’t it? The competitors’ “claim”about what cannot have happened is used to support a flat assertion about what must havehappened.

(A), (E) The evidence in each choice is factual (the results of the judging in (A), and (E)’sstatement about the late filers). But the stimulus evidence is a claim that some party ismaking, and an argument that lacks that element cannot be parallel to the original.

(C) The conditional “If the census is to be believed” has no parallel in the stimulus, nordoes the movement from small groups (married men vs. married women) to large (all menvs. all women).

(D) is, yes, based on a claim that isn’t factually proven. But we are looking for a claim thatsomething could not have occurred as evidence for a conclusion about what must be thecase. So the evidence in (D) falls short, and the conclusion that the product is safe for othercreatures isn’t like the stimulus conclusion either.

• When you can’t symbolize the terms of a Parallel Reasoning stimulus algebraically,step back from the content to characterize the nature of the argument under analysis(as we do, above, in the sentence beginning “The essence of the logic...”). Then youcan compare its nature to that of the choices, rejecting the ones that deviate as youmove swiftly to discovering the truly parallel choice.

Page 30: Prep Test 6

LSAT PREP ________________________________________________________________ LSAT Test VI Explained: Section II

28 © K A P L A N

6. (C)The structure couldn’t be laid out more clearly: High-tech medicine is driving costs up(conclusion); the cataract example “illustrate[s] why” (evidence). The example goes on toindicate that now that technology has made the surgery easier and cheaper, more and morepeople are having it done, thus driving up the total cost. “Hang on a second, back up,” youmight have said to yourself. “Didn’t the author just say that cataract surgery is cheapernow?” Indeed he did, and yet he concludes that total costs are up. It has to be true, then, as(C) says, that whatever money is saved due to the lower price of the procedure is offset bythe increased number of patients opting for the surgery.

(A) That the surgery was “not always effective” 10 years ago is a far cry from saying that“few” surgeries were effective. For all we know, there were still many effective surgeriesback then, even though the surgery is more effective today.

(B)’s scope is the future—what is “likely to” happen in the long run—but that cannot beinferred from this argument set in the past and present.

(D) We are explicitly told that what is driving up costs is not the elderly’s demand forsurgery, but high-tech medicine. Cataract surgery—admittedly an ailment of the elderly—is used as an example, not identified as the main culprit.

(E) seems to run counter to the fifth sentence: the passage explicitly indicates that thesurgery is cheaper now, which strongly suggests that it’s more affordable now than it wasten years ago.

• Be on the lookout at all times for “au-contraire” choices—those that are the oppositeof what the author believes or what the question is looking for.

7. (E)Four of the choices weaken the explanation of why so many more people are having theircataracts done; therefore the right answer must either strengthen it, or be outside the scope.And remember what that explanation is: they’re going for the surgery, says our author,because the technology makes it less painful, more successful, and cheaper. There’s reallyno way to pre-phrase here—we simply have to check the choices, looking for the one thateither strengthens the argument, or is irrelevant to it.

(A) asserts that there are more people now. More people could lead, in general, to morepeople needing or wanting surgery, which on its face could explain the increase.

(B) strongly suggests that the prevalence of the disease may have increased. An increasedneed for cataract surgery would certainly explain an increase in its incidence.

(C) asserts that there are more elderly now, and the author admits that elderly people areprone to cataracts. As with (A), this change in the demographics would suggest anotherreason besides improved technology for the increase in surgeries.

Page 31: Prep Test 6

LSAT PREP ________________________________________________________________ LSAT Test VI Explained: Section II

© K A P L A N 29

(D) The fact that more people don’t have to pay for the procedure could also reasonablyaccount for an increased number of people opting for it. This, too, would thereforechallenge the author’s argument that the increase in cataract operations is due to thebenefits conferred by the new technology.

(E) is too vague to challenge the author’s explanation. Are these unlucky patients victimsof the new technology or the old? We simply don’t know. Since (E) doesn’t differentiatebetween unsuccessful surgery then and now, and doesn’t even tell us what percentage ofsurgeries fail, we have to consider the consequences of unsuccessful surgery in general tobe irrelevant to the specific line of reasoning here. (E)’s therefore the one we want.

• Study each Logical Reasoning stem carefully, before reading the stimulus. Often thestem will give you clues as to what to read for, and sometimes (as here) even do partof your work for you. Q. 7’s stem tells us the author’s purpose (to explain) and anaspect of the scope (increased cataract operations). This gives us a head start onanalyzing the argument.

8. (A)The paradox, simply put, is that employers don’t want to hire workers who once servedunder non-compete agreements, even though the courts have pretty much ruled that thesepromises not to work for a competitor aren’t binding. There must be something about theprocess that new employers find off-putting . . . and (A) describes it: the costs and badpublicity of going through a lawsuit, even one that (the author implies) the new employeris likely to win. (A) would explain the new hirer’s reluctance.

(B), if anything, would deepen the paradox, by narrowing the applicant pool to competitors’employees, making it even more puzzling that firms would balk at hiring them.

(C)’s focus is on the old employer, the one who imposes the non-compete agreements. Butthe paradox has to do with the new hirer.

(D), like (C), has its focus on the wrong parties. That most current employees are loath togo to a competitor is irrelevant to the issue of those (whatever their number) who do wantto switch firms, but can’t because of the new firm’s qualms.

(E) suggests that firms would be eager to hire employees who have establishedrelationships with clients while working at other companies, which doesn’t explain whycompanies would be hesitant to hire employees bound by non-complete agreements whenthey know those agreements aren’t binding.

• Once you’ve restated the paradox, think about alternative possibilities that theauthor may not have considered. Doing so is often the fast track to the right answerin a Paradox question.

Page 32: Prep Test 6

LSAT PREP ________________________________________________________________ LSAT Test VI Explained: Section II

30 © K A P L A N

9. (C)Both women are concerned with how to make a country admirable. Inez’s point—incontrast to Mary Ann’s point that what matters is that a country be strong—is that a countrybe moral. The evidence? If its actions are moral, then it is admirable—and she points outthat countries can be strong but most UNadmirable indeed. All of this presupposes that acountry’s moral compass can be evaluated—or, as (C) puts it, that one can assign moralvalue to countries’ actions. If (C) is false, if morality is not a scale on which countries can beweighed, then Inez’s argument for judging countries on a moral basis is blown away.

(A) Inez’s argument is conditional: If a country is morally good, then it’s admirable. Doesthere have to be even one country that meets Inez’s standard? No. For all we know, everycountry falls short, but this wouldn’t alter Inez’s argument, so (A) need not be assumed.

(B) Inez doesn’t deny that a country can be both strong and moral; her counterexamplesmerely indicate that strength is not a necessary condition of admirableness, so herargument does not rely on (B).

(D) Citizens’ beliefs are never mentioned by Inez, so she needn’t be assuming anythingabout them.

(E) merges the issues of morality and strength together in an ominous assertion that themorally good country had better use its strength to get other countries to shape up, or else.This is a far cry from Inez’s rather idealistic, even genteel point of view; her argumentsurely doesn’t presuppose this.

• Be on the lookout for the classic wrong answer types. Citizens’ beliefs (D) are outsidethe scope, and therefore can have nothing to do with the necessary presuppositionhere. Similarly, (E) confounds the whole issue by altering the argument fromindividuals assessing the morality of a country to countries imposing theirmoralities on others—”by whatever means necessary,” no less! It’s distorted, it’sextreme; it can’t be the assumption here.

10. (E)Something’s not right. All of John’s friends say they know a really healthy 40-year smoker(that’s what they claim to be true), but John doesn’t know one, and “he is not unique amonghis friends in this respect.” That quoted phrase—a statement of fact—means that at least oneof them does not know such a smoker. Since they all claim they do, at least one of John’sfriends is not telling the truth. That’s (E).

(A) The proven liars are among John’s friends, who may or may not smoke themselves.

(B) Intent isn’t the issue, nor is exaggeration. The issue is the contradiction between whatJohn’s friends profess to be true, and what is true.

(C) For all we know, each of John’s friends who professes to know a really healthy 40-yearsmoker is talking about a different smoker! They don’t all have to be referring to the sameone.

Page 33: Prep Test 6

LSAT PREP ________________________________________________________________ LSAT Test VI Explained: Section II

© K A P L A N 31

(D) “Most” is what kills this one. It only need be true that one of John’s friends is lying.

• The LSAT tests your ability to interpret words and phrases correctly. “Notunique...in this respect” means that there are some who are like him in this respect—that is, some who also don’t know a healthy 40-year smoker. This paraphrase of thetext is the key to getting the point.

• “Some” literally means “at least one.” That’s all it can be inferred to mean.

11. (B)“. . . [I]t is imperative that . . . ,” “must be able to . . . ,” “is . . . unable to . . . ”: Phrases likethese are sending you a signal that the author’s main interest is in necessary conditions—conditions that are required for some result to occur. Sentence 1 says that a necessarycondition of democracy’s survival is the average citizen’s informed opinions, and sentence2 narrows that to focus on opinions about science. Sentence 3 comes along to assert that inthe face of today’s scientific developments, the average Joe or Jane cannot absorb enoughinfo to hold such informed opinions. But, as we’ve already seen, without such opinions,democracy can’t survive; so it has to be true, as (B) has it, that today’s scientificdevelopments threaten democracy.

(A) The author never mentions scientists, so we cannot infer from her remarks that theyhave the responsibility to attack the problem. For all we know, other groups such asscience writers or educators can help the public form meaningful scientific opinions.

(C) Democracy’s survival requires that the average Joe or Jane, not that every Joe or Jane,hold informed opinions on science. Also, the phrase “scientifically literate” isn’t used inthe stimulus, so we can’t be sure that it applies.

(D) is a classic extreme choice, in that it blows things way out of proportion. The stimulusimplies that extremely advanced scientific knowledge threatens democracy, not, as (D)would have it, that any scientific knowledge is bad for democracy. The notion that sciencemay be getting a little difficult for the average citizen to comprehend doesn’t lead to thenotion that the least scientific knowledge would yield the best democracy.

(E) tests necessity/sufficiency confusion in a classic way. Citizens having informedopinions on science are necessary for democracy’s survival but aren’t necessarily sufficientfor it. In other words, democracy could be destroyed (for other reasons) even if everysingle citizen becomes scientifically informed and opinionated.

• When a condition necessary for a result does not occur, the result cannot occur.Keep this principle in mind, as it is at the heart of most LR questions that hinge onnecessity.

Page 34: Prep Test 6

LSAT PREP ________________________________________________________________ LSAT Test VI Explained: Section II

32 © K A P L A N

12. (E)“However” signals where the “discrepancy” that we are to resolve is to be found. The topicis the use of the fossil record near a former glacier to date the appearance of a warmclimate, and here’s the apparent paradox: While the insect record suggests that right afterthe glacier melted, a warm climate appeared, the pollen record in the same area suggeststhat the warm climate came much, much later. (E) does nothing to explain this difference indates: beetles may be older than “many” plants, but there could be plenty of other pollen-bearing plants that are as old or that even predate the beetles. (E) is therefore irrelevant tothe seeming contradiction in the stimulus, and is thus the choice we seek that does not helpto explain the discrepancy.

The four wrong choices all paper over the dating discrepancy in different yet valid ways:

(A)’s solution is that the fossils were cold-weather beetles, who returned to the area whilethe climate was still cold. End of discrepancy: The pollen record now reigns uncriticized.

(B) explains how plants might take longer to appear in a warm area, and thus suggests whythe plant fossil record might give the impression that the warming occurred later than itactually did.

(C) Since (C) shows how beetles could survive in a barren post-glacial area before thearrival of plants, it explains why the beetle fossils antedate the plant fossils.

(D) indicates that the plant record may not accurately reflect the real date of new plantgrowth following the melting of the glacier, which suggests that the pollen record may notnecessarily accurately reflect the onset of the warm period. If this is true, the discrepancyin the fossil record is no longer surprising.

• In a “discrepancy” or “paradox” question, be sure to paraphrase the paradox insimple terms in your head before you plunge into the answer choices.

• Previewing the stem can help you in many different ways. In the case of thisEXCEPT question, the stem tells you that the answer you seek is the one that’s notrelevant to the discrepancy in question, or the one that in fact even deepens themystery.

13. (E)The paragraph lays out a chain of cause and effect that begins with the overall point madein the first sentence: Using these new “clean-coal” technologies in factories could reducepollutants and acid rain. How? By cutting a lot of noxious emissions (sentence 2), which inturn would reduce noxious ozone production in the troposphere (sentence 3). You don’thave to understand what all of this technical terminology means; you simply have to seethat in the author’s mind, the factory technology strategy mentioned in sentence 1 couldlead to a reduction of tropospheric ozone in sentence three, choice (E). Put another way: Ifone thing (new tech in factories) could lead to another (reduced emissions), and the second(reduced emissions) does lead to a third (reduction in noxious ozone), then it’s fair to saythat the first could lead to the third.

Page 35: Prep Test 6

LSAT PREP ________________________________________________________________ LSAT Test VI Explained: Section II

© K A P L A N 33

(A) The author is way too vague on which of the pollutants contributes to acid rain for (A)to be a valid inference. For all we know, sulfur dioxide has little or no impact on the acidrain phenomenon.

(B) Nitrogen pollutants are all that affects noxious ozone formation, as far as this passage isconcerned; sulfur dioxide may have little or no role. (And anyway, it says that noxiousozone is formed in the troposphere—don’t think there are too many factories there.)

(C) distorts the statistic that these new technologies could cut sulfur dioxide emissions by80%. For all we know, even one per cent of sulfur dioxide is unsafe.

(D) The concept of “careful design” is never raised here, and is so vague that it cannot bepart of a proper inference. More importantly, (D) is about designing new factories, whilethe stimulus is about reconfiguring existing ones.

• In Inference questions, be on the lookout for terms (like (D)’s “careful design”) thathave no bearing on the text. Such choices are invariably wrong. (See also (B) and (C)in Q. 18, below.)

14. (D)There are lots of characters here, so make sure you keep them straight. First, there’s JoshuaSmith, who wrote a book. Then there’s a book editor who criticizes the book as“implausible.” Finally, there’s the author of the stimulus, who criticizes the editor’scriticism, stating that it is “unwarranted.” Got all that? Now, on what grounds does thestimulus author feel the editor’s criticism of the book is off base? Each of the incidents inthe novel “could very well have happened”—in other words, each incident is “plausible.”But just because the individual events ring true doesn’t mean that they will do likewisewhen strung together. The author is committing what is sometimes called the “part towhole” fallacy, the assumption that that which is true of the parts must be true of thewhole. (D) states the flaw; the “given characteristic” mentioned is “plausibility.”

(A) and (C) both key off the sideswipe at the editor in sentence 2, but that catty reference tothe editor’s previous off-base criticisms is just background information, not central to theauthor’s rebuttal. (Note that that entire phrase could be dropped from the paragraph withno damage to the logic.)

(B) criticizes the author for relying on people’s judgment concerning whether anindividual incident is plausible. Granted, we might be mistaken concerning what isplausible. But the fact remains that even if each event in the book is plausible by itself, thebook as a whole could be implausible. So (B) fails to address the central issue here.

(E) The evidence isn’t necessarily relevant only to those who already believe theconclusion. One could accept the evidence that each of the novel’s incidents is plausible,yet reject the conclusion that the entire novel is likewise.

• Use Kaplan’s PrepTest explanations to learn about the various types of faulty logicperennially tested on the LSAT. Take careful notes on them all. For another exampleof part:whole weakness, see Section III, Q. 4.

Page 36: Prep Test 6

LSAT PREP ________________________________________________________________ LSAT Test VI Explained: Section II

34 © K A P L A N

15. (C)The conclusion is that the skills needed for creative research are teachable and learnable,but not a single word in the evidence directly mentions skills or creative research. Allwe’re told is that Thomson taught tons of physicists of great distinction, so two keyassumptions are at work here. First, the author must assume that Thomson’s distinguishedstudents do, in fact, have creative research skills—but this isn’t listed as an answer choice.Second, she must assume that at least some of those distinguished students lacked creativeresearch skills before they met up with Thomson. After all, if they were all fully creativeresearchers beforehand, then Thomson’s influence wouldn’t support the conclusion,would it? That second assumption is what we get in (C).

(A) The argument could work even if (A) were false, even if Thomson were relativelyunknown and all his students came out of his own home town. Thus the Kaplan DenialTest proves that (A) isn’t a necessary assumption.

(B) needn’t be assumed, because even if Thomson taught some people who were neverrecognized for any accomplishment, the author still has the eminent students to supporther argument.

(D) “Other fields” have no role in this argument. (D) raises an unwarranted comparisonbetween other fields and physics, and the validity of the argument does not depend on anysuch comparison.

(E) isn’t a necessary assumption either, because Thomson’s successful students may be rareexceptions. The author’s point is that creative research skills can be taught, and thepossibility that few successful researchers are taught by renowned scholars (the denial of(E)) doesn’t hinder that argument.

• Be watchful for terms mentioned in the conclusion that are absent from the evidence(such as “skills” and “creative research” here). Recognizing such scope shifts isoften the key to spotting the author’s necessary assumption or nailing down theargument’s logical flaw.

16. (E)Only the phrase “explanation of the difference described” marks this as a Paradoxquestion, but that’s enough; we are indeed asked to find a choice that resolves an apparentcontradiction contained in the argument. We are told of a seemingly inconsistent use ofwater power by the Romans: extensive in the boonies, none at all around the cities. Thatsure sounds odd, yes? But a pre-phrase is certainly possible here: We can search for anchoice that speaks to city conditions that would somehow render water power unnecessaryor even detrimental, and the latter is what (E) provides. The possibly inflammatory effectof introducing water power in and around cities (social unrest) would certainly argueagainst its use, and does help us to better understand the situation described in thestimulus.

(A) is evidence of Roman water expertise (which the author grants in the first sentence) butfails to address why such water use would stop at the city limits.

Page 37: Prep Test 6

LSAT PREP ________________________________________________________________ LSAT Test VI Explained: Section II

© K A P L A N 35

(B) just says that in the cities, water was available all year round with some “seasonalvariation.” But so what? If anything that would deepen the paradox, making it even morepuzzling that all that available water wasn’t used by the Romans to make energy.

(C) is a wash: “Water power was vulnerable, but damage from sabotage could be readilyfixed.” That’s all well and good—but we’re right back where we started: Why wasn’t it usedin and around cities?

(D) It would stand to reason that areas not using water power would have some energysource. But that still doesn’t answer the question “why not use water power in the citieswhen it’s used in the more outlying regions?” All of the wrong choices leave this centralquestion unanswered, while (E) provides a satisfactory explanation.

• Read unfamiliar question stems carefully, so that you can recognize familiarquestion types that are somehow “in disguise.” Recognizing this one as essentially aParadox question helps us to discard the choices that have no bearing on thesurprising circumstance.

17. (B)The stem is a little confusing and requires careful interpreting: It means “On what groundsdoes the reviewer criticize the book she is reviewing?” Well, she does two things: object toone of the three alleged energy storage methods (electricity), and claim that at least threeothers have been left out. That means the book is guilty of sins of both commission andomission—which is more or less (B) in a nutshell: One of the methods, electricity, is wrong(inaccurate), and others have been left out (the list is not exhaustive).

(A) distorts the main issue, which is not “Which storage method is most ‘basic’?” but israther “What are the basic storage methods?”

(C) The use of energy is (C)’s concern, but not that of the book under review nor thereviewer herself.

(D), (E) The effectiveness of energy storage methods is wholly outside the scope. Note (E)’stwo additional problems: (E), like (A), seems to believe that the issue is “Which method ismost basic?”; and (E) creates an unwarranted distinction between electricity and the others.

• Many wrong choices deviate from the scope—a healthy reminder to keep theauthor’s scope in mind throughout your reading of each stimulus and set of choices.

Page 38: Prep Test 6

LSAT PREP ________________________________________________________________ LSAT Test VI Explained: Section II

36 © K A P L A N

18. (E)Previewing the stem, we see that we need to discern the author’s main point. When theauthor begins by saying that something “is no mystery,” it’s a good bet that his point willbe the mystery’s solution, and so it is here. Those unfamiliar with the term “figurativepainting” must figure out that it means “recognizable images,” and you can do so bycombining the first two sentences. The author figures that in the 1970s, people wanted toempathize with paintings, and puzzling over abstract art didn’t afford this opportunity; heequates “lack of realism” with abstraction in the final sentence, and asserts that abstract artleft people alienated. So what explains the revival of figurative art in the 70’s? Evidently,the author believes the public’s taste is the solution to the mystery, choice (E).

(A) Quite the contrary: if abstract art did reflect reality, by the author’s own logic peoplemight have been drawn to it. His whole point is that abstract art is divorced from reality—which may help explain why the 70’s public resented it.

(B) is a little vague—technically, the idea of “traditionalism” is not mentioned at all. Buteven if we interpret “traditional” to mean the recognizable realistic paintings the authorbelieves that people prefer to abstract paintings, one would have to conclude from theargument that people wanted to see such subjects portrayed in traditional ways, notnontraditional ways as (B) would have it.

(C) The argument is about what the art-viewing public wants to see, and why; not whichform of art is more difficult to create. Talent plays no part in the argument, which is why(C) is far from the author’s main point.

(D) The passage doesn’t lead us to believe that the public can’t understand the theoreticalbasis of abstract art. It’s just that, in the 1970s, they didn’t want to. They wanted an art closerto home.

• A “main point” or “conclusion” question works like any other inference: It must betrue based on the text.

19. (E)The argument we are to weaken is that the high wages paid to Valitanian (where do they getthese names?) elected officials have corrupted them. The evidence for this charge: Why,simply that high wages always attract corrupt individuals, those most interested in liningtheir pockets. To the author, that is a truism, but she provides no real evidence against theseValitanian elected officials. Therefore, we can weaken the indictment against them if wecan somehow show them to be above petty financial matters. And that’s just what (E) does.If (E) is true—if most of these officials could make more money elsewhere—then the cordbetween this evidence and this conclusion is severed; the officials may still be corrupt, butnot for the reason proposed. In the face of (E), the author would have to come up withalternative evidence for the conclusion; and that is proof positive that (E) has weakened theargument.

Page 39: Prep Test 6

LSAT PREP ________________________________________________________________ LSAT Test VI Explained: Section II

© K A P L A N 37

(A) Spending one’s own cash in the hopes of cashing in big upon election might beconsidered a reasonable investment, even for the corrupt money-hungry politiciansportrayed by the author. Nothing in (A) weakens the argument; candidates could spendsome of their own money on their campaigns and still be mostly in politics for the highsalaries.

(B) Term length being unmentioned in the argument, we cannot conclude what effect, ifany, a four-year term would have on officials’ integrity.

(C) The argument is about whether money attracts worse candidates, not more candidates.Their number is irrelevant.

(D) may well describe most of Valitania’s politicians, in which case it provides somesupport for the conclusion.

• Study this argument, choice (E), and explanation carefully. Together they provide anexcellent textbook example of what it means to weaken an argument.

20. (C)This adviser has an interesting and unusual take on freedom of speech: It’s not just right,it’s smart. How so? Because keeping ideas silent benefits no one, while airing ideasencourages the good ones and reveals the bad ones so that they can be rebutted. A pre-phrased answer to “What’s the method?” should demand a choice that combines these tworationales—morality and practicality—and that line of thought must lead one to (C).

(A) Circularity means that the conclusion is merely assumed to be true; in other words,that the evidence and conclusion are functionally identical. Not so here: The authorprovides independent evidence (sentences 2 and 3) for the conclusion (sentence 1).

(B) Au contraire, the author goes beyond the “right” of free speech to its wisdom andpracticality.

(D) Based on the last sentence, the author evidently takes for granted that the truth canreadily be suppressed, by force. (He just doesn’t think that doing so is smart.)

(E) By saying that free speech is a government’s “only rational policy,” the author isexplicitly dwelling in the real, not the ideal. And the negative outlook of this choice(cannot be achieved) doesn’t match the tone of the argument.

• Notice that the stem here tells you the author’s purpose before you even read theparagraph. That makes your reading job easier. Get in the habit of analyzing eachstem first!

Page 40: Prep Test 6

LSAT PREP ________________________________________________________________ LSAT Test VI Explained: Section II

38 © K A P L A N

21. (D)The evidence is that under free speech, good ideas blossom and bad ideas revealthemselves, thus making them easier to shoot down. A “smart” government will allow freespeech in order to protect itself against danger and promote order for all. Look for a choicethat speaks to these issues, and you’re likely to latch onto (D) quickly. If true freedom ofspeech reduces the risk of revolution, wouldn’t a government be smart, as the authorcontends, to adopt it?

(A) That most would tolerate limits on free speech doesn’t ensure that a small but angryminority might not cause big, big trouble. Besides, if anything, (A) seems to run counter tothe spirit of the argument: The author wants the government to adopt freedom of speech,while (A) offers a possible reason why the government might get away with curtailing it tosome extent.

(B) distorts the issue of “rationality”: The author feels that adopting free speech is therational thing to do, but the rationality or irrationality of the government’s response todangerous ideas is one step removed from the argument. We can guess that this authorwould probably hold that a rational response to dangerous ideas is more likely whenspeech is free...but that’s another argument for another day.

(C) Since the author never argues that free speech is the only, or the most important,freedom for a government and a people, (C) neither strengthens nor weakens his logic.Other basic rights are outside the scope of the argument.

(E) is one argument for the irrelevancy of free speech. Au contraire!

• Sometimes the second question of a two-parter is the easier of the two. Here, theabstract choices of Q. 20 suggests that this may be the case, and you may havebenefited from jumping right to Q. 21 first. It’s okay to handle one while putting theother off for later.

22. (D)We’re asked, What’s the flaw in the plan?, so we had better understand its goal beforestudying its specifics. The drama workshop wants (says the last sentence) to offerscholarships only to those with the best-rated auditions. To achieve this, they’ll endow thetop 10% of locals and the top 10% of nonlocals with scholarships. Whoa!, you should say toyourself—a scope shift! The breakdown of auditioners into local/nonlocal has nothing todo with audition quality, and hence the two criteria could possibly clash. For instance, whatif the local talent is so far superior to that of the nonlocals that the auditions of the second10% of locals (the ones right below the 10% who will get scholarships) get higher ratingsthan any of the top nonlocal talent? It’s possible, right? And if it happens, then theworkshop will be undercutting its stated goal, because some people with worse ratings(the nonlocals) will get scholarships over better rated auditioners. (D) has its finger on thescope shift.

(A) The terms of the plan are: talent, local vs. nonlocal applicants, and scholarship offers.(A) mentions none of these, in favor of: enrollment, other programs, and admissionapplications. So (A) is outside the scope for many reasons.

Page 41: Prep Test 6

LSAT PREP ________________________________________________________________ LSAT Test VI Explained: Section II

© K A P L A N 39

(B)’s reminder of the subjectivity and hence fragility of drama auditions doesn’t take awayfrom the plan’s effectiveness, because all of the auditioners will be equally prey to thepossibly inaccurate results. The plan concerns what will happen when all the evaluations,accurate or otherwise, are in.

(C) is a variation on the “can vs. should” scope shift. Someone’s need (or lack of need) forfinancial aid has nothing to do with the wisdom of offering that person aid. This is the scopeshift that (C) commits, however, and not the one that the author commits.

(E) Fairness-schmairness. The issue is why the workshop may be cutting its own throat inits plan to offer scholarships to the best-rated auditioners. Fair or unfair, only (D) explainswhy the plan may not work. In any case, who says that the plan favors nonlocals? For all weknow, the plan may result in fewer nonlocals being offered scholarships.

• Watch out for authors’ shifting the scope from the terms of their evidence to those oftheir conclusion. But don’t pick answer choices that shift the scope!

23. (E)We’re asked for an inference, so let’s see how the argument is constructed. The argumentproceeds from general comments about the author’s criteria for novels to a specific claimthat Peter Lee’s San Francisco novel “passes [her] test.” Sentences 1 and 3 make it clear thatit is a necessary condition of her full enjoyment that the novel earn her trust by the author’sknowing a city “at least as well as [she does].” Thus, since Lee’s novel passes her test, itwould have to be true that, in the opinion of the reviewer, Lee does know San Francisco atleast as well as she does; and (E) just restates that.

(A) The reviewer’s enjoyment of a good novel is increased by her trust. We can’t infer fromthis that she enjoys any novel whose author she trusts.

(B) essentially reverses the terms in sentence 3—something we are not allowed to do. Thereviewer might well trust novels that are set in unfamiliar locales. The logic here startsfrom a conditional statement: “When I read a novel set in a city I know well...” We thereforedon’t know what it takes to gain the reviewer’s trust in all other cases—it’s very possiblethat she trusts a novelist whose book is set in a city alien to her, so (B) cannot be inferred.

(C) All we can infer about Lee’s first novel is that—since it, too, passed the reviewer’stest—it was set in a city the reviewer knew well, and Lee’s knowledge of it at least matchedher own. But that city may or may not have been S.F.

(D) As we saw above in (B), we don’t know how the reviewer feels about books set in citiesunknown to her.

• Remember that the answer to an Inference question must have a high truththreshold—it MUST be true based on the text. Accept nothing less.

Page 42: Prep Test 6

LSAT PREP ________________________________________________________________ LSAT Test VI Explained: Section II

40 © K A P L A N

• The word “when,” like “if,” sets up a conditional statement, which means thateverything that follows is based strictly and only on the particular state of affairsdesignated by the conditional. Here, everything follows from a novel in questionbeing set in a city the reviewer knows well. How does the reviewer respond to novelsset in cities she doesn’t know well? We can’t possibly know, which is why (B) and(D) fail as proper inferences.

24. (C)We’re given a principle of conduct, and asked for the real-life scenario that best matchesup to it. The issue is, “When is it morally justified to benefit from harming someone else?”And the principle cites a two-part necessary condition, signaled as usual by “only if”: thevictim’s awareness that the behavior could be harmful, and his/her consent. We must seethat in the absence of such awareness and/or consent, the benefits accruing from the harmwill not be morally justified. Having done that analysis, we can then proceed withconfidence to the choices and see how they stack up:

(A) ignores the whole issue of awareness and consent, focusing instead on issues (falsestatements; preventing harm) that the principle never mentions. Besides, Sonia attempts tobenefit, but does not—she gets stuck in detention anyway—so the morality test doesn’tapply here; no one here benefits from harming another.

(B) brings in the concept of accidental behavior, and again ignores the awareness andconsent issues.

(C) Here ya go. Max lacked awareness of the potential harm of the experimental drug.Thus a necessary condition for the moral justification in Wesley’s using the results islacking. He harmed Max without informing Max of the risks. Thus, according to theprinciple, it is not morally justified for Wesley to benefit from the harm he inflicted onMax, and so the judgment in (C) is right on the money. For the record:

(D) Roger’s mother consented to the operation, and the wording seems to suggest that, as adoctor, she’d be aware of the risks. Since this situation meets the two criteria for moraljustification, we cannot use the principle at hand to conclude that Roger was morallywrong to benefit from the transplant.

(E) argues that taking the profits from James is unjustifiable, but that can’t be correct here:James is the one trying to benefit from having done harm, and the principle in the stimuluscertainly wouldn’t support it in this case (who, after all, consents to being defrauded?).Denying James the profits would be in line with the principle in the stimulus, but (E) saysthat denying James is UNjustifiable, which conflicts with the principle.

• Principle questions usually present a real-life situation and ask us to pick the oneabstract principle, out of five, that applies. Q. 24 reverses this. But the approach is thesame: Think topic and scope. Look for the one and only choice that matches up wellin all particulars; toss out choices that enter new territory.

Page 43: Prep Test 6

LSAT PREP ________________________________________________________________ LSAT Test VI Explained: Section II

© K A P L A N 41

25. (C)Because of its stem’s wordiness and the formality of all the prose, this question seemstougher than it actually is. The stem, first of all, alerts us that it’s Parallel Reasoning, andthat what we’ll be looking at is (1) a government’s policy, (2) that policy’s intent, and (3)some relevant circumstance. All we need to do is match up the stimulus’ (1), (2), and (3)with those of the right answer. In the stimulus, (1) is farm subsidies, (2) is the goal ofbolstering domestic yield, and (3) is the effect of that policy: more intensive farming thatexhausts the soil and hurts domestic yield. In short, then, we want a self-defeating policy: apolicy that goes after one goal but in the end achieves the exact opposite. That’s what weneed to look for among the choices:

(A) would work better if the policy of subsidizing theaters to enhance tourism actuallyrepelled tourists. But all (A) is saying is that theater is largely irrelevant to tourists’ purchasedecision.

(B) would work better if the policy of restricting imports to keep domestic producers aliveactually drove them into bankruptcy! Instead, (B)’s policy works even better than planned:It doesn’t just keep the domestic producers alive, but helps them rack up big profits.

(C)’s policy of building a strong army to avoid armed conflict actually brings about armedconflict, which is needed to keep the army strong in the first place. This is the self-defeatingpolicy we need—a policy that aims for one thing but brings about its opposite. For therecord:

(D) would work better if the policy of reducing taxes to stimulate investment actually killedinvestment. But (D) just goes on to say that the chancy nature of investment—a totallyoutside issue—may affect things. Not parallel.

(E) comes so close, with a policy (traffic laws), an intent (safer travel), and the oppositeeffect achieved (less safety). Where (E) goes wrong is that it’s not the policy that brings aboutthe opposite result, but the outside factor of population growth that increases trafficvolume. (E) misses the major element present in the stimulus and in (C): a policy that’sdesigned to bring about a result but that directly brings about its opposite. Here, we dohave the opposite of an intended result, but the policy itself is not to blame, so we cannotlabel the policy “self-defeating.”

• Don’t be deterred by a lengthy question stem: If you take the time to take it apart, itcan tell you a lot about what you’re looking for and make your job much easier.

• In non-algebraic Parallel Reasoning questions, abstract from the text and try toparaphrase the general structure as simply as possible. In a case like this, don’tthrow in words like “farming,” “subsidies,” and “soil exhaustion”—these are theparticulars, and the right answer won’t be about these things. But “self-defeatingpolicy” is perfect—it describes in a general way the gist, the action of the stimulus,and the very thing we are to look for in the right answer.

Page 44: Prep Test 6

42 © K A P L A N

SECTION III:

LOGICAL REASONING

Page 45: Prep Test 6

LSAT PREP _______________________________________________________________ LSAT Test VI Explained: Section III

© K A P L A N 43

1. (A)According to the first sentence, a condition necessary for a flower to be worth growing in agarden is that it smell sweet. Some roses, we’re told, have no scent at all, sweet orotherwise; they, like all roses, provide stunning color, but are scentless. All in all, then,there must be some flowers—namely the scentless roses—that provide stunning color butaren’t worth growing. That’s just what (A) says.

(B) The only scentless flowers mentioned are “some roses.” Other types of flowers mightbe both scentless and visually drab.

(C) contradicts the passage’s necessary condition. Only flowers that smell sweet are worthgrowing in a garden, so there can be no flowers worth growing that are scentless.

(D) is possible only: It’s possible that some sweet-scented roses might still not be worthgrowing in a garden. But it’s equally possible that all sweet-smelling flowers, roses orotherwise, are worth growing. No way to tell.

(E) Nothing in the stimulus makes stunning color a requirement for cultivation in agarden.

• The word “only” should always grab your attention. It signifies the always-tested,always-important concept of necessary conditions.

2. (B)The conclusion—signaled by the phrase “That this is true is shown by”—is an assertion ofcause and effect: Using money causes a decline in a civilization. But the evidence allamounts to correlation only: When money (of either type, real or paper) was used, Westerncivilization declined. A counterexample will suffice as a “refutation” of the alleged causeand effect, and that’s what (B) provides—a specific example in which the assertion in thefirst sentence does not hold. With the author’s example supporting the conclusion and (B)’sdenying it, we’re left at a stalemate.

(A) People’s preferences are outside the scope here and don’t have anything necessarily todo with what’s good or bad for a civilization.

(C), if anything, strengthens the author’s case by explaining how money may cause, or atleast contribute to, a civilization’s decline.

(D) asserts an unwarranted distinction between gold and paper, which the author lumpstogether as being bad for civilization. Also, of course, we have no idea how the exchangerate figures into this equation.

(E), if anything, supports the argument that money—especially paper money, which iswhat most employees get—is bad. By choosing to compensate employees with goods—thatis, with “articles of intrinsic value”—instead of “worthless” money, the mentality of theemployers in (E) is at least consistent with the attitude of the author.

Page 46: Prep Test 6

LSAT PREP _______________________________________________________________ LSAT Test VI Explained: Section III

44 © K A P L A N

• Always be alert to references to cause-and-effect . . . and expect many authors tomistake it for correlation.

3. (D)The conclusion is the last sentence’s prediction that importing predators into the U.S. willstop the increase in U.S. fire ants, and the evidence is the past success of this strategy on fireants in Brazil. Four assumptions connect evidence and conclusion, and some students willtry to locate the “odd man out” and proceed to (D) directly by noticing its scope shift. Thegoal of the strategy is to protect the environment by stopping the increase in the fire antpopulation, not to restrict the spread of the ants to northern states. Even if (D) is false, evenif the use of predators wouldn’t prevent some northern spread of the fire ants, the strategystill might be useful in reducing the ant population overall—which is its goal! Since (D) canbe denied with no damage to the logic, it is not a necessary assumption here.

Other students will, in a situation like this, choose to locate the four assumptions, and herepre-phrasing is certainly in order:

(A) You may very well have wondered whether the plan has any major drawbacks. Theauthor—eager to sic those predators on the ants—doesn’t seem concerned aboutdrawbacks, so he must be taking for granted that the “cure” won’t be worse than the“disease” (A).

(B) Perhaps the most obvious assumption here is that what worked in Brazil will work inthe U.S., which is (B).

(C), (E) Finally, the author assumes that the plan will work without a hitch, which leads usto the remaining two choices. If the U.S. ants are able to kill the predators, then the planwill fail; so the author is assuming that that won’t happen (C). The plan will also fail if therate of increase is so huge that even the predators can’t slow it; so the author is assumingthat that won’t occur either (E).

• An “All of the following are assumptions EXCEPT” question tells you two things: (1)the argument must be pretty vulnerable, what with four “gaps” between the evidenceand conclusion, and (2) the right answer will be either outside the scope of theargument (as (D) is here), or contradictory to it.

4. (B)Wanting people to buy the pesticide, and afraid that they won’t if they fear its toxicity inplants, the manufacturer argues that there’s less of the toxic stuff in the pesticide than ineveryday mouthwash, “ounce for ounce.” That last phrase is the one that should grab you.OK, so the stuff is safe ounce for ounce. But how many ounces are we talking about here?How much of the stuff does a person take in? If, as (B) says, the quantity of toxins that aperson ingests from plants exposed to the stuff vastly exceeds the quantity ingested frommouthwash, then the manufacturer’s “ounce-for ounce” defense of the product is ill-taken.

Page 47: Prep Test 6

LSAT PREP _______________________________________________________________ LSAT Test VI Explained: Section III

© K A P L A N 45

(A) fails to address the issue of the toxins in the pesticides. Until that’s addressed, themouthwash remains a basis of comparison only, and the comparison of mouthwash toother cosmetic products is irrelevant to the argument that the pesticide is safe.

(C) The clarity of toxicity warnings is irrelevant to the safety of the product itself, and tothe ad claim made by the manufacturer, which are after all what the question is askingabout.

(D) At best, the rate at which toxins become harmful is irrelevant to the question of themanufacturer’s claims here. At worst, (D) slightly supports those claims, by speaking to therelative safety of the pesticide product as compared with mouthwash.

(E) Government regulation and pesticide awareness may be relevant to the overall issue ofpesticide use, but not to this specific argument about this specific product.

• What’s true of a part of the whole isn’t necessarily true of the whole itself—a conceptoften tested on the LSAT. Ounce for ounce, the pesticide stacks up well withmouthwash, according to the manufacturer. But that’s only part of the story. Howmuch of this stuff is actually ingested from plants treated with the pesticide is thereal issue here.

5. (D)We’re looking for an inference that can properly be derived from the stimulus. There arefour research companies estimating the population of the same three cities. For two of thecities, all the estimates agree; for the third city, the estimates vary widely. How come? Thedifference is that the first two cities are stable while the third city is “growing rapidly.”There’s one thing we can deduce with certainty up front: Since the companies’ estimatesfor the rapidly growing city vary so wildly, someone’s got to be wrong. That’s not amongthe choices, however, so as is often the case with Inference questions, we’ll have to evaluatethe choices, looking for the one that’s supported best by the information at hand.

(A), (E) Unwarranted distinctions. No surveys of smaller cities are made here (A), nor isany mention made about current vs. future estimates (E). Neither of these comparisons isinferable here from the information given.

(B) The paragraph stops short of mentioning, let alone evaluating, the uses to which thedata might be put. The “usefulness” of the data is outside the scope.

(C), if anything, works against the implicit general idea that it’s difficult to determine thepopulation of a rapidly-growing city. But even if you didn’t see that, you should have seenthat there’s no way to infer anything about the rate of population change here—it verylikely may fluctuate.

(D) Since all four estimates are roughly the same for the stable cities in this study, it’sreasonable to say that these companies’ population estimates for stable cities in general willlikely coincide. And if they do, then their reliability must be roughly the same, whetherthey’re all right on, or all way off.

Page 48: Prep Test 6

LSAT PREP _______________________________________________________________ LSAT Test VI Explained: Section III

46 © K A P L A N

• Notice how correct choice (D) is highly qualified, by virtue of the word “likely.” Tosay that the companies would definitely be equally reliable in their estimates ofstable cities might be a bit of a stretch; but to say that it’s likely is perfectly in therealm of believability—and perfectly in line with the phrasing of the stem.

• Of course, (D) also comes to the fore because of the problems with all of the wrongchoices. You need not be fully in love with (D) in order to conclude that among thefive choices, it’s the one that’s best supported by the text.

6. (E)Next up we need a fact that accounts for the disparity between the companies’ agreementon the population estimates for the two stable cities, and their profound DISagreement onthe estimate for the growing city. There has to be some factor that will account for thedifference, but it may not be easy to pre-phrase what they’re looking for here. Moststudents will therefore evaluate each choice in turn:

(A) Finding “uniformity” in the growing city’s demographics deepens, rather than solves,the mystery of why these companies all disagree on how many people live there.

(B) The reason for the one city’s growth is irrelevant to the disparity in survey results.Whatever the reason, there must be something more going on to get such a curiousoutcome.

(C) It is not necessarily tougher to estimate the number of young people vs. older people,so the difference in average age, while being perfectly consistent with the data in thestimulus, doesn’t at all speak to the odd survey results.

(D) describes the methodology employed on the growing city. But since (D) doesn’tcontrast the methods used on the stable ones, it cannot explain the difference in data. Infact, (D), like (A), may even deepen the mystery by showing that all the companies usedroughly the same method for estimating the population of the growing city, and still cameout with widely divergent results.

(E) does the trick. That the companies’ methodologies differed in their estimates of thegrowing city as opposed to the stable ones would help to explain why the results on theformer were all over the map while the results on the latter jibed well. When surveymethods are the same, one will not be surprised that their results are the same. Whensurvey methods differ, it’s much less of a surprise to find disagreeing data.

• As you study your practice questions, pay just as much attention to the explanationsof the ones you got wrong as the ones you got right. Every question—and, therefore,every Kaplan explanation—has the potential to teach you something for test day,whether or not you got the question right in practice. Don’t shortchange yourself.

Page 49: Prep Test 6

LSAT PREP _______________________________________________________________ LSAT Test VI Explained: Section III

© K A P L A N 47

7. (A)There are two scope shifts going on here, one fairly obvious, one more subtle. The obviousone is the movement from the motorcyclist evidence to the conclusion about horsebackriders; the two riders are linked in that they can both suffer serious head injuries. But therisk of injury per se is not (according to the author) why some jurisdictions have requiredmotorcyclists to wear helmets, nor why the author wants other areas to do so and why shewants to expand the requirement to horseback riders. The real reason is cost: in the second,third, and fourth sentences the author emphasizes that wearing helmets reduces injury andhence reduces the huge financial drain on taxpayers. Seeing the evidence shift the scopefrom injuries to cost should lead you to (A), because without (A) we cannot be sure that ahelmet law for horseback riders is a good idea for the same reason that it works formotorcyclists. Unless and until we learn that horseback riders’ head injuries cause afinancial drain, we cannot agree that achieving “similar cost reductions” is a properjustification for a horseback riders’ helmet law akin to the motorcycle-helmet law.

(B) speaks to the difference in likelihood of injury alluded to in the last sentence, but theargument doesn’t depend on some justification for this difference. What the argument doesneed is a statement of similarity: a similarity between the costs of the two types of non-helmeted injury.

(C) rightly hits on the issue of cost, but goes astray elsewhere: It makes an unwarranted andunhelpful distinction between head injuries (of all types) and other types of injuries. Those“others” are outside the scope here.

(D) Fatalities and their prevention are not part of this argument; injuries and their cost are.

(E) By putting citizens’ safety above all other considerations when looking at helmet laws,(E) goes against the grain of the author’s apparent belief that reducing the cost to taxpayersis the central justification for the enactment of these laws.

• This is a great example of why you should always study the question stem beforereading the stimulus. You know that the conclusion deals with helmets forhorseback riders, and hence you can put all the motorcycle evidence in its propercontext.

8. (D)The senator’s position—that no work of art can be obscene, and hence that the world of arthas nothing to do with protecting the public—is attacked by means of counterexamples,works of art that people call obscene. His response? “[I]f [they] really are obscene then theycannot be works of art.” But that statement is just the contrapositive of the meaning of hisoriginal remark. “No work of art can be obscene” means “IF a work of art, THEN notobscene.” Reverse and negate both terms and you get the senator’s statement in the lastsentence. So what’s wrong with the reasoning? The senator provides no independentevidence as to the nature of obscenity and the nature of art—he defends the originalprinciple by invoking its contrapositive, that is, its logical equivalent. So he is guilty of“circular reasoning” or, as (D) puts it, assuming the truth of that which he is trying toprove.

Page 50: Prep Test 6

LSAT PREP _______________________________________________________________ LSAT Test VI Explained: Section III

48 © K A P L A N

(A) The senator uses no emotional language or appeal. He is trying to persuade by logic,but he fails because he begs the question by restating his conclusion as if it offered moresupport. That’s what (D) is saying.

(B) Au contraire, his points are logically equivalent (as mentioned above, contrapositives ofone another)—not contradictory.

(C) would be true if he alleged (however absurdly) “Art is not obscene because I am asenator and what I say goes.” There is, of course, no such assertion of authority here.

(E) Again, his appeal is to his own conclusion; he veers off not into the irrelevant, but intothe redundant.

• “Circular reasoning,” or “assuming one’s conclusion,” is when one’s evidence andconclusion are functionally identical. Since such arguments show up on the LSATonly occasionally, those terms are usually mentioned in wrong choices rather thancorrect ones.

9. (A)Why must Hastings have been disloyal? Because if his firing was justified, then he musthave been either incompetent or disloyal, but the former is ruled out. That seems to leaveonly the latter, “disloyal”—but all of that rests on the key issue of justification, whichmerely appears in an “if” clause in the argument. To proceed from that conditional “if”clause to her categorical conclusion, the author must be taking for granted that that “if”clause is confirmed—that Hastings’ firing was in fact justified. The Denial Test proves it: Ifcontrary to (A) Hastings’ firing was not justified, then we have no evidence upon which tobase a finding of disloyalty.

(B) Hastings’ rank, high or otherwise, goes unmentioned, so the author need not beassuming anything about it.

(C) The scope of this argument is Hastings and Hastings only; the author need not assumeanything about others.

(D) According to the author, someone justifiably dismissed could be loyal—as long ass/he was incompetent. Since (D) flies in the face of the second sentence, it certainly can’t bethe assumption we seek here.

(E) commits the “fallacy of affirming the consequent,” i.e. reversing the if and then terms ofsentence two. The author doesn’t do so; she simply takes for granted that her “if” clause isaffirmed . . . which, again, is (A).

• Scope is a concept just as important in Logical Reasoning as it is in ReadingComprehension. Always be aware of the author’s scope, and avoid choices thatdepart from it.

Page 51: Prep Test 6

LSAT PREP _______________________________________________________________ LSAT Test VI Explained: Section III

© K A P L A N 49

• Don’t overlook the little words! Noticing that the second sentence begins with theword “if,” and understanding the conditional situation that follows from this word(which the author goes on to forget about or ignore), are the keys to getting thispoint.

10. (D)Were you able to spot the common logical flaw in this reasoning? Hopefully yourecognized the author’s confusion of necessary and sufficient conditions, and searched thechoices looking for a similar mistake. It’s helpful to translate the first sentence into if-thenform: IF you don’t answer questions, THEN you’re not a competent doctor. So answeringquestions is necessary to being a good doctor. Fine. But then the author goes on to say that“Since my doctor answers my questions fully, she must be competent.” Uh-oh! The authormistakes a condition necessary for being a good doctor (answering questions) for one that’ssufficient for being a good doctor. Being a good question answerer is required of gooddoctors, but that doesn’t mean that’s the only requirement—a doctor can answer questionswell and still be a bad doctor for other reasons. So we’re to find a situation in whichnecessity and sufficiency are confused in a similar manner. Check out the choices:

(A) Close, but no cigar: Being accustomed to making compromises is necessary togrowing up in a large family. But being accustomed to making compromises doesn’tguarantee that one grew up in a large family; others can certainly pick up this habit as well.So (A) seems to be in line with the original, but veers off at the last moment: (A) wouldexhibit the same necessary/sufficient confusion of the original if it concluded “thereforeMeredith MUST have grown up in a large family.” But the tentative nature of (A)’sconclusion has no parallel to the stimulus.

(B) is proper logic. Jeanne, like any other opponent of the issue, must be ill-informed. Notflawed, so not parallel.

(C) Paul’s situation contradicts the first sentence, which said that all those who like musicnever miss a performance. He likes music, yet blew off the performance. Curious. We cancharacterize this scenario as “an exception to a general rule,” which is not the action of thestimulus argument. There’s no confusion of necessary and sufficient conditions in (C), so(C) is not parallel to the original.

(D) If someone works two or more jobs, that person cannot find a balance between careerand personal life. Is there an element of necessity here? Yup: Working fewer than two jobsis a necessary requirement for balance. Maggie meets this condition—she has only one job.And just like in the original, the author of (D) treats this necessary condition as sufficient inconcluding that Maggie can therefore achieve balance between career and personal life.Not necessarily! She’s cleared this one hurdle, but for all we know working fewer than twojobs is not the only thing necessary to achieving such balance. Same confusion, sameresult—(D)’s the winner here.

Page 52: Prep Test 6

LSAT PREP _______________________________________________________________ LSAT Test VI Explained: Section III

50 © K A P L A N

(E) There’s necessity here as well: It’s necessary to be cool and easygoing (or however wewish to describe “not hot-tempered and strong-willed”) to succeed. Jeremy fails to meetone of the conditions right off the bat (he is strong-willed), so (E) deviates already on thiscount. There can be no confusion of a necessary for a sufficient condition if Jeremy doesn’tmeet the necessary condition in the first place. (E) is flawed, sure: We cannot concludefrom the mere fact of Jeremy’s strong will that he won’t succeed in the business, because weknow nothing of his hot temper. This flaw, however, is far from the flaw in the original.

• In “Find the Parallel Flaw” questions, often the easiest choices to eliminate are onesthat exhibit valid reasoning. Your response to (B) should have been “that soundsreasonable . . .”—which should have enabled you to cut (B) right away.

• Make sure you’re crystal clear on the issue of necessity vs. sufficiency. Often thisconcept will be tested in Inference and Logical Flaw questions. Here, it constitutedthe pattern of flawed reasoning that we had to spot among the choices.

11. (A)Since Stevens’ article was accepted for publication, it must have met sentence 1’s twonecessary conditions for publication—submitted prior to March 6 (which we’re told), andwritten by a certified psychoanalyst (which we’re not told but must be true anyway,because otherwise the passage would be self-contradictory). We are told that Stevens“publishes frequently in psychoanalytic literature,” but don’t be fooled—that’s not thesame as saying that he is, in fact, a certified psychoanalyst. But he has to be—choice (A)—because it’s a necessary condition for the article’s publication. (Note that (A) would beequally correct, and maybe even a little more precise, if it read “Stevens is a certifiedpsychoanalyst.”)

(B) distorts sentence 2: The fact that Stevens publishes frequently in psychoanalyticliterature doesn’t tell us in which publications his articles appear. Sure, this article wasaccepted by the Journal, but we can’t conclude from the info given that the Journal“frequently accepts Stevens’ articles.” For all we know, this acceptance is a rare exception,and most of Stevens’ articles appear elsewhere.

(C) isn’t necessarily what the author means by “certified.” For all we know Stevens iscertified but not an authority; or perhaps he’s an authority in one field rather than many.

(D) This particular article might not be one of the articles that the Journal actively solicited.If sentence 1 stated that the Journal only publishes solicited articles, then (D) would beinferable. But it just says that the Journal often solicits articles, so (D) isn’t inferable.

(E) The passage’s scope is this one article, and the Journal’s publication criteria. Audienceinterest is outside that scope.

• If two conditions are necessary for a conclusion to be met, and we are told that theconclusion is met, then both of the conditions must be true. That’s what “necessity”means.

Page 53: Prep Test 6

LSAT PREP _______________________________________________________________ LSAT Test VI Explained: Section III

© K A P L A N 51

12. (C)Determining the point of the argument hinges on one tiny phrase: “so-and-so is like arguingthat the yeti etc.” To be sure, all the rest of the evidence concerns the Himalayan yeti, butthat’s all in the service of “so-and-so,” the analogy that precedes the yeti material. The pointis that just as the absence of documentation for the yeti doesn’t prove that it doesn’t exist,the absence of documentation for European/East Asian trade in a certain time framedoesn’t prove that it didn’t happen. It’s the historical trade issue that the author isinterested in; the yeti is just used as analogous evidence. (C) is right on.

(A) The author applies the term “scientific confirmation” to the yeti example only, andanyway it doesn’t really apply to a situation in which one is trying to prove whether tradeexisted between two world regions.

(B) The author never indicates that we need evidence from both sides; we can’t even besure that the author agrees with (B), and so (B) cannot be the main point of the argument.

(D) focuses on disproving the idea that the author seems interested in proving or, at least,keeping alive (that these two areas traded during the Middle Ages). The author, by use ofthe analogy, tries to say the opposite: Just because there are no records, that alone doesn’tdisprove the existence of the trade.

(E) merely compares the relative amounts of evidence for the two analogous allegedphenomena, when what the author’s trying to do is keep alive the possibility of tradebetween Europe and East Asia in the face of the absence of evidence. (E) gives too muchprominence to the place of the yeti example in the argument; as discussed above, it’s thereto shed light on the trade issue, and is thus not part of the main point itself.

• There’s a good chance that somewhere on your LSAT, most likely in LogicalReasoning, they’ll test whether you know what an analogy is and how to properlyapply it. Be watchful.

13. (E)Here, the yeti example really has nothing to do with the trade issue; it’s just a parallel. Thusthis argument shouldn’t be hard to counter. We want a choice indicating that the absence ofhard evidence for European/East Asian trade is significant, that it damages the tradehypothesis; and that’s exactly what (E) does. There do exist written records that shouldmention such trade had it actually gone on, but don’t. That’s pretty strong counter-evidence: (E) provides a situation in which the absence of references to an event has aprofound impact on determining whether it did in fact occur. If (E) is true, we’re less likelyto believe that the absence of evidence is no big deal—it now seems like a really big deal,and the author’s argument appears to be sunk.

(A) If anything, the author would cling to (A) as support for his idea that the absence ofwritten reports doesn’t deny that European/East Asian trade existed.

(B) has much the same problem as (B) did in Q. 12: The argument doesn’t hinge on the needfor TWO-WAY proof that this kind of trade went on, so appealing to that issue cannotstrengthen or weaken the logic.

Page 54: Prep Test 6

LSAT PREP _______________________________________________________________ LSAT Test VI Explained: Section III

52 © K A P L A N

(B) Just because East Asia had records and Europe didn’t doesn’t mean that trade did ordid not exist between the areas. We have no idea what the records said, and so their mereexistence tells us nothing.

(C) The nature of European/East Asian trade is utterly irrelevant unless it has an effect onwritten records of such trade—which (C)’s facts do not.

(D) We might have expected a choice or two dealing only with the yeti, ready to trap thosewho failed to see the real topic and scope of the argument. Hope you weren’t one of them.As mentioned above in Q. 12, the yeti is used only to illustrate the real point, so even ifproof positive were found of the yeti’s existence, its impact on whether Europe and EastAsia traded during the Middle Ages would be zilch.

• Arguments by analogy are weak by definition, since one’s opponent can either justdeny that the analogy is valid or come up with a counter-analogy of his own. Lookfor the LSAT writers to use this point as the basis for questions.

14. (B)This Parallel Reasoning stimulus lends itself nicely to an algebraic treatment. If theeconomy is in recession (X), then demand is low (Y), and if demand is low, then interestrates are low (Z). Combining these statements, we get “if X, then Z,” and the authorconcludes with the contrapositive of this—“if not Z [if interest rates aren’t low] then not X[no recession].” So this is valid logic, paralleled exactly by (B), where X = the ready fish, Y= cooked all the way through, and Z = white. Both arguments conclude correctly that if theZ is not so, the X cannot be so.

(A) starts with an X and Y term that complement each other: a full restaurant means a fullparking lot and vice versa. There’s no subsequent “Z” term brought in, so (A) isn’t parallel.

(C), too, only gives us an X and Y term: X = flying by flapping wings, and Y = warm-blooded. Then we get negations of these terms: “Gliding” is used as “not X,” and “cold-blooded” is, of course, “not Y.” Just like in choice (A), there’s no true “Z” term.Furthermore, the phrase “if they flew at all” has no parallel in the original anywhere.

(D) Here, we could reasonably say that X = pleats, Y = doubled material, and Z = havingnone left for the top. The conclusion is that if X (if you want pleats) then Z (won’t make thetop). This is valid reasoning, but as you’ll recall, the original argument doesn’t formallycombine the statements in this way—it concludes with the contrapositive of thecombination of the first two statements. (D) fails to go on to draw this final conclusion. Itwould be parallel if, instead of the last sentence, it finished with “if you have material leftfor the top, then you didn’t want to put in pleats.”

(E) has its X, Y, and Z terms right, but concludes that if Z (if the party loses) then X (theforecasters were right). This is classic faulty logic: The forecasters could be wrong, and yetthe governing party could still lose for other reasons besides inflation. Bad logic, and notparallel.

Page 55: Prep Test 6

LSAT PREP _______________________________________________________________ LSAT Test VI Explained: Section III

© K A P L A N 53

• Not every Parallel Reasoning stimulus lends itself to algebraic translation; but whenit does, the technique can be very effective.

15. (B)Sentence 1 is factual only: The potato crop in Rosinia is down about 40 million tons from 20years ago. Sentence 2 is the argument, the explanation as to why this is so: The researcherswho have failed to come up with new high-yield potato strains are at fault. Well, if thefailure to produce higher-yielding crops has caused a decrease in the harvest, then the oldstrains of potatoes must not have produced the yields that they used to. Check out theDenial Test: If (B) is false, if the old strain of spuds are producing as well as ever, then thelack of higher-yielding strains wouldn’t by itself explain the harvest’s drop; it would onlyaccount for a failure to increase the harvest. In short, if (B) is false, blame for the situationmust be placed elsewhere than on the researchers. Since (B), when denied, weakens thelogic, (B) as written is a necessary assumption.

(A) goes too far. Though the researchers haven’t produced new high-yield strains so far, theauthor needn’t assume that they never will.

(C), by implying that a researcher working selfishly could still come up with somethingthat would benefit Rosinia, somewhat undermines the argument that the researchers’ failureis due to their misplaced motives.

(D) weakens the logic in a different way from (C), by suggesting that the differences inharvest size might be a natural fluctuation, rather than a steady reduction to be blamed onresearchers.

(E) is outside the scope, since the argument never refers to funding or governmentinvolvement. Nothing need be assumed about this.

• As you read an argument, keep stepping back from it and asking: What’s the natureand purpose of each sentence? (See above.) Doing so will keep you sharp andanalytical, and will prevent you from becoming too wrapped up in the content.

16. (E)Some students believe that the story of an army stopping to drink a lake dry must haveactually occurred, because there’s evidence that water-based life died off there at the sametime as the alleged “big gulp.” Well, heads up here: Just because there is an indication thata certain effect (here, the drying up of the lake) took place, we cannot use it as evidencethat any one particular cause for that effect—whether far-fetched or plausible—must havetaken place. Any number of things could have decimated the water-based life at thatlakesite (drought brought on by climactic changes, for example), and this legend is but oneof them. Put another way: The archaeological evidence may very well suggest that thewater disappeared at the time the legend has it; that is, that the effect described in the text iscorrect—the lake did go dry. But that doesn’t lend credence to the text’s assertion that amillion thirsty warriors slurped it away.

Page 56: Prep Test 6

LSAT PREP _______________________________________________________________ LSAT Test VI Explained: Section III

54 © K A P L A N

(A) The conclusion is not a generalization; it is an assertion about the truth of one specificevent at one specific locale.

(B) No “counterevidence” is mentioned. What the students are “ignoring” is the fact thatno one cause for an effect is any better than another cause, until persuasive evidence iscited.

(C) A self-contradiction is when one asserts two things that cannot both be true. Not sohere. The story of the army drinking the lake could be true (if unlikely); it doesn’t contradictthe little field evidence discovered. Besides, (C) contradicts the spirit of the stimulus—ifanything, the students are accepting, not rejecting something (although what they’reaccepting is better described as a legend than a “hypothesis”).

(D) For all we know, historians have substantiated the existence of an army of Pavonianenemies at the time. And the location is certainly real, too, as verified by the archaeologists.

• Always be wary when a stimulus describes an allegation of cause-and-effect.Causality is very tough to prove, and hence is the basis of many reasoning errors(and hence is at the heart of many LSAT questions). Ask yourself: MUST the effectbe the result of an alleged cause, or could something else be at work here?

17. (A)Before weakening the skeptics’ suggestion we need to know what they’re skeptical about,and it’s the timeline that has been posited between human activities and the carbon datingat each level of the Pennsylvania rock shelter. What’s controversial here is the deepestcarbon sample. If this is a timeline of human activity, then the earliest humans must haveinhabited the site approximately 20,000 years ago. It’s that that the skeptics take issue with:since that dating conflicts with what has been believed about the arrival of humans inNorth America, they suggest that contamination is to blame, that the carbon simply seepedout of nearby coal deposits into the lowest level (and, thus, that the presence of carbon thatdeep has nothing to do with human activity at all).

Now, there is no evidence that carbon contamination occurred; it’s simply a plausiblealternative theory that the skeptics have cooked up because they don’t like the one at hand(i.e. that humans lived in North America 20,000 years ago). So if we can render thatalternative theory IMplausible, then we will have shut the skeptics up—and that’s just what(A) does. According to (A), if such carbon contamination had occurred, it could only havedone so by contaminating the upper levels too, which would fly in the face of the excellentcorrelation between carbon samples and human activity above. (After all, since theuppermost layers have been dated to the present, they clearly have not been contaminated.)In other words, (A) says that in the absence of contamination evidence up above, it’s totallyunlikely that contamination could have occurred below—which, again, effectively silencesthe skeptics.

(B)’s suggestion of an occasional weakness in the carbon-dating technique doesn’tnecessarily affect this application of the technique.

Page 57: Prep Test 6

LSAT PREP _______________________________________________________________ LSAT Test VI Explained: Section III

© K A P L A N 55

(C) at best is irrelevant (human use of coal is not part of either the original theory or that ofthe skeptics), and at worst is an au-contraire choice, since the skeptics are trying to deny thepresence of humans 20,000 years ago and (C) seems to support that denial.

(D) merely speaks to the accuracy of the carbon dating, but what is at issue are theinferences that can be drawn from it with regard to humanity.

(E) is certainly au contraire. This is in line with the skeptics’ view that this Pennsylvaniasite’s early carbon sampling is an anomaly, and not an effective challenge to earlierestimates of when humans appeared in North America.

• What makes a difficult LR question difficult is often the juxtaposing of severalpoints of view. Make sure you clearly formulate the gist of each point of view beforeyou start analyzing the answer choices, and always keep in mind which viewpointyou’re asked to critique, strengthen, weaken, etc.

18. (A)A categorical conclusion: There’s no scientific reason to reject astrology, despite thewidespread opinion that those who believe in astrology are credulous and unscientific. Asproof, the author offers many people of scientific brilliance in years past who tookastrology as fact. Well, this represents a scope shift from what was once believed true towhat now is believed true, and (A) puts its finger on it. That astrology was accepted at anearlier time does not mean, with the advances of science that followed, that it has to beaccepted now. That which the earlier scientists took to be true may long since have beendisproved.

(B) argues that there’s no point in arguing for or against a “controversial” opinion, whichwould end all debate on everything. Talk about specious: Here’s a specious point of view.

(C) Even if (C) is true, and the great scientists who once accepted astrology were onlythose influenced by Western science, that doesn’t affect the argument’s validity. Astrologyeither is scientific or it isn’t, for all cultures.

(D) The argument assumes that modern Western science makes people reject astrology, so(D) makes no sense.

(E) is beside the point. The argument is only concerned with scientific reasons to accept orreject astrology. It is certainly true that nonscientific reasons have been left out, or“overlooked,” but, as nonscientific reasons are outside the scope here, that omissiondoesn’t take anything away from the argument.

• Scope shifts between evidence and conclusion are common, and when they occurare almost always at the heart of the question you’re being asked. Always be on thelookout for them!

Page 58: Prep Test 6

LSAT PREP _______________________________________________________________ LSAT Test VI Explained: Section III

56 © K A P L A N

19. (D)It’s easy to get lost in all the verbiage until you see the structure underneath, which turnsout to be like a “logic tree” or “decision tree”—it’s simply a matter of following the formallogic trail, and remembering, from previewing the stem, that we’re looking for somethingthat must be false. Sentence 1 raises the issue: Will McConnell decide to run against Lutz?Sentences 2 and 3 raise two possibilities: Either Lutz has a big war chest (sentence 2) or hedoesn’t (sentence 3). If the former, McConnell will bow out. If the latter, she’s a step closerto getting into the race, but needs to investigate Lutz, and sentences 4 and 5 state the onlytwo outcomes: Either Lutz has skeletons in his closet (sentence 4), which will improve herchances of winning, in which case McConnell will run; or else, Lutz is clean (sentence 5),and she won’t.

In essence, then, there is only one scenario under which McConnell will run against Lutz:He lacks a big war chest AND he has scandalous skeletons in his closet. Thus (D)contradicts the passage, since it presents a case in which Lutz’s record doesn’t enhanceMcConnell’s chances (which is code for “he has no scandals”), yet she runs anyway.According to the passage, she explicitly would NOT run under these circumstances.

(A) would be true if—emboldened by Lutz’s lacking a big war chest—McConnell went onto investigate but found no scandals in his record. She would then choose not to run.

(B) is perfectly consistent with the only scenario under which McConnell would opposeLutz.

(C) could be true if Lutz had a big campaign war chest, in which case McConnell wouldchoose not to oppose him even before it got to the point of checking out his record.Remember the order of events in her decision tree: Scandalous items means she’ll run onlyafter she’s determined that he’s not loaded.

(E), of course, would close the matter immediately. Lutz’s big money would (sentence 2)cause McConnell to opt out of the race, so (E) jibes perfectly with the conditions in thestimulus.

• In a case such as this, where the right answer contradicts the passage, the four wrongchoices are consistent with it. It is therefore your call as to whether you want to goafter the odd man out directly, or keep throwing out consistent ones until you locatethe one contradictor. Either approach is efficient.

Page 59: Prep Test 6

LSAT PREP _______________________________________________________________ LSAT Test VI Explained: Section III

© K A P L A N 57

20. (C)The conclusion (which, as we already know from the stem, is flawed) is utterly categorical:There is just no way that psychotherapy, as some have alleged, can be a form of coercion.Why not? Because of a dichotomy the author sees: Coercion restricts people’s choices,while the goal of psychotherapy is to make it easier for people to make choices. But is thecontradiction as clear-cut as the author believes? What if—contrary to its stated goals—psychotherapy in some cases has the end result of restricting the ways in which people runtheir lives? What if some psychotherapists coerce their patients into following only acertain set of choices? In either case, psychotherapy might be just as restrictive and“coercive” as its critics allege. Look at it this way: If the second clause of the secondsentence read like so, the argument would be stronger: “...and people who go throughpsychotherapy always leave with an enhanced ability to make choices.” But then theevidence would be focusing on the way psychotherapy takes place and not merely its goal.As (C) says, losing sight of the possible difference between psychotherapy’s goals, and itspractices and actual results, is where the author goes wrong.

(A) In no sense is the author stacking the deck by “redefining” terms “unfairly.” (Howwould we be in a position to distinguish fair and unfair, anyway?) She simply defines eachterm in the statement she is rebutting so that she can put forth her position. She can do noless in order to make her rebuttal, and does so straightforwardly, as far as we can see.

(B) Whatever its goals and whatever their number, if the end result of psychotherapy iscoercion, then the argument remains weak. The number of goals of psychotherapy is not theissue here, so (B) sidesteps the problem.

(D) The author does seem to believe that the means and ends of psychotherapy areidentical—in fact, another way correct choice (C) could have been written is that the author“fails to see that the means and goals need not be the same.” But that is different fromstating that the means are “justified.” Justification for a practice is not the author’s purposehere; it’s rebutting a point of view.

(E) If psychotherapy were proven to be a form of moral coercion, then a defender ofpsychotherapy might follow (E)’s lead and show that coercion might not always be a badthing. But our author preempts that tactic by trying to show that the term “coercion”doesn’t even apply to psychotherapy.

• When examining an argument that the stem tells you is flawed, ask yourself: Whatcould be done to the argument to improve it? The answer to this question may helpyou identify the flaw as written.

Page 60: Prep Test 6

LSAT PREP _______________________________________________________________ LSAT Test VI Explained: Section III

58 © K A P L A N

21. (E)Joel defines “myth” as a narrative conveying the community’s wisdom, and goes on todeny that myth can exist today because of the absence of such communal wisdom. Giselledisagrees, pointing out an example (the very word might have made (E) tempting) of amyth that exists in the modern world and is accepted by one particular community(doctors). Giselle acknowledges that part of her counterexample doesn’t apply to Joel’sdefinition—the myth of the body-as-machine isn’t a narrative—but she argues that it is amyth nonetheless. For medical people, at least, the Myth of the Machine is a powerful andgenerally accepted truth; it conveys that community’s traditional wisdom. (E) has it right:This counterexample, this modern myth, calls into question Joel’s claim that a myth mustbe a narrative.

(A) distorts Giselle’s science-based example into “a scientific explanation.” And the“problem” is not one of literary theory, but of reality: Does myth—can myth—flourishtoday?

(B) is tempting because Giselle does seem to use an analogy (the body is like a machine) asthe basis of her modern myth, but her overall position is contrary to Joel’s, not analogous toit.

(C) comes closer to characterizing Joel’s statement than Giselle’s. He speaks of “traditionalwisdom” and “the modern world,” while she draws no distinction between past andpresent.

(D) Giselle’s rhetorical question is meant to suggest that the community of medicalspecialists would agree with her that the body-as-machine qualifies as a real myth. In noway does she imply that Joel is one of their number.

• Whenever answer choices are written in abstract terms, be sure to take a stab at pre-phrasing an answer before getting caught up in all of the abstraction. Even a pre-phrasing that’s not too close to the real answer can still help you to avoid wrongchoices.

22. (A)The conclusion, first of all, is a comparison. On the issue of which classification will betterreflect the scientific significance of some fossils, the author votes for a recent one over theone proposed by the discoverer of the fossils, Walcott. But the author doesn’t (as sheshould) discuss the superiority of this recent classification over Walcott’s; she simplyargues that as a member of the science elite, Walcott’s view would simply mimic the viewsof the establishment. As choice (A) has it, this is drawing a conclusion about a positionbased on the source of the position rather than on the position’s actual merit.

(B) The first piece of evidence cited is that Walcott was a big shot in the scientific elite,which is more “verifiable” than (B) lets on. The second piece is a conclusion (sentence 3)that follows from the first, and it too could be verified: One could verify whether Walcottsimply confirmed what established science deemed true.

Page 61: Prep Test 6

LSAT PREP _______________________________________________________________ LSAT Test VI Explained: Section III

© K A P L A N 59

(C) See the analysis of the evidence in (B), above. These premises don’t contradict eachother; the second follows from the first.

(D) Denying that “not X” can be true in order to prove that “X” is true is questionablelogic, yes, but not the issue here. The evidence concerns Walcott’s standing in the scientificcommunity and ignores his science; that’s the problem.

(E) jumbles past and present. That Walcott’s classification was made in the past is notrelevant to the main objection, which is that his views were simply mimicking those of thescientific elite. That “category” (that of the elite) could just as readily apply to Walcott thenas now.

• In Logical Flaw questions, determine whether the author is addressing the merits ofa position or merely rejecting it out of hand.

23. (D)Judith so clearly lays out the issues that we don’t really need to read Anthony’s remark atall (though we should, of course). She doesn’t deny his conclusion, but she rejects the wayhe’s reached it. What she is pointing out, of course, is the difference between correlationand causation. Anthony asserts that since there’s a high correlation between heroin andprior marijuana use, the latter must lead to the former. But as Judith points out, there’s aperfect correlation between drinking water and heroin use, yet it would be idiotic to arguecause-and-effect there. She shows, then, the silly result that can occur when one mistakescorrelation for causality. (D) puts this in slightly more formal terms.

(A) She quibbles not with the accuracy of his statistic, but the use to which it’s put.

(B) distorts Judith’s use of “absurd.” She doesn’t derive anything from his conclusion;rather, she comes up with a parallel argument illustrating the weakness in his reasoning. Itis his method of deriving his conclusion (assuming that correlation necessarily impliescausation) that Judith believes can lead to absurd consequences.

(C) The safety of heroin use isn’t the issue. Judith’s objection is methodological, notpharmaceutical. She is using the notion that drinking water does not lead to heroin use toillustrate the absurdity of Anthony’s reasoning, so (C) is way off base.

(E) goes too far by inferring that because she takes issue with Anthony’s use of a statistic,Judith must believe that no causal connection can be supported by statistics. She impliesnothing as sweeping as this; (E) is too extreme to be Judith’s method here.

• The LSAT regularly tests the ability to distinguish between correlation (an assertionthat X and Y tend to occur together) and causation (an assertion that X causes Y). Bealert for this.

Page 62: Prep Test 6

LSAT PREP _______________________________________________________________ LSAT Test VI Explained: Section III

60 © K A P L A N

24. (B)Here’s a different misuse of statistics. All of the percentages compared are parts of the samewholes: total revenues two years ago vs. now. Compared with two years ago, the carmakers’ share has dropped 15%; parts suppliers’ share has gone up 5% and servicecompanies’, 10%. But these are all just parts of the pie. The initial question—have autorevenues declined?—has to do with the overall pie itself, and no matter how the shareshave varied over the last two years, the data never address the central issue of what hashappened to the overall total. (B)’s wording is a little oblique, but speaks to the scope shiftthat the author commits: Since all the parts must add up to 100%, only information aboutthat 100% is relevant. Regardless of the increases cited, revenue for the overall industrymay very well be declining, and (B) helps point out why the increases cited can’t be usedto explain away the rumored decrease in overall industry revenues in the way the authorintends.

(A) The source of the statistics is irrelevant to whether the statistics are being properlyused.

(C) demands a reason for the change in shares, but that’s beside the point. Has the overalltotal dropped or hasn’t it, and does the change in shares support the case one way or theother?

(D) speaks to the relationships among the different sectors of the industry. Again, noimpact on the basic question under consideration.

(E) The argument is about revenues only, not profits.

• LSAT wrong answers often accuse the motives behind, or accuracy of, statisticaldata. Right answers tend to focus on the use to which the data are put, especiallywhen the flawed reasoning is based on a faulty application of statistics.

25. (E)First, a proposal is mentioned: to extend the U.S. school year in the Japanese and Europeanmanner. Then an objection is cited: Ditching summer vacation would violate a longtradition. Finally, the conclusion—the “objection misses its mark”—and the evidence: the“tradition” began because farmers needed kids to be free to work during the summer. Thereal tradition to follow, the author says, would be to follow the needs of the economy inscheduling school (which in turn might warrant changing the school year, as is beingproposed).

The principle that backs up this logic is going to focus on the reasons underlying atradition (i.e. the reason that the objection “misses its mark”), and that’s (E), which firmlydistinguishes between a tradition itself and the reasons for the tradition in the first place.Following (E) to the letter would mandate going beyond the sheer 3-month summervacation to the reason it was first instituted, which in turn would support the author’s viewthat the proposal wouldn’t defy tradition in the way that the opponents claim.

Page 63: Prep Test 6

LSAT PREP _______________________________________________________________ LSAT Test VI Explained: Section III

© K A P L A N 61

(A) encourages the changing of policies and traditions that conflict with more pressingneeds. But the author’s point is that, correctly understood, lengthening the school yearwouldn’t really violate a tradition at all. In the same way, (B), (C), and (D) all intend toattack, or at least to limit, the importance of tradition, but the author is trying to show thata policy change (eliminating summer vacation) is not out of line with tradition—the realtradition as she explains it in the paragraph. The anti-traditionalist sentiment of all fourwrong choices hurts, rather than helps, the author’s mission here.

• The author’s purpose is a very important and much talked about issue, but studentsoften lose sight of it in “the heat of battle.” Always try to determine why an author ismaking an argument and is making the claims she does. Doing so will help you,whether your job is simply to analyze the logic or to fiddle with it in some way.

26. (C)“What is tradition?” That’s the author’s question. She argues that it should be understooddifferently, based not upon the specific practice (three months of vacation) but rather uponits intent (economic need). (C) paraphrases this nicely.

(A) The author accepts that schools are shut down for three months. That’s not in dispute.

(B) Au contraire, the author expressly uses historical practices in formulating her approachto the problem under discussion.

(D) The author isn’t really attacking her opponents personally, as (D) implies. At most, sheis asserting not that they don’t care about tradition but that they have misunderstood it.

(E) sort of sounds like the author’s approval of the proposal. But “the rest of theindustrialized world” goes way beyond simply “Japan and Europe.” And she stops shortof “demonstrating” that tradition would demand revoking summer vacation. To do so,she’d have to provide evidence as to the economic need for summer school, but shedoesn’t.

• Sometimes when a stimulus generates two questions, the second is easier to answerthan the first. Get in the habit of scanning both stems and deciding which ordersuits you better. Take control of your exam, to every extent that you can.

Page 64: Prep Test 6

62 © K A P L A N

SECTION IV:

LOGIC GAMES

Page 65: Prep Test 6

LSAT PREP _______________________________________________________________ LSAT Test VI Explained: Section IV

© K A P L A N 63

Game 1—Petworld(Q. 1-6)

The Action: Immediately upon seeing “14 animals. . . in four separate cages,” you have tofigure that your job will be to place the former in the latter—to carry out, in short, agrouping game of distribution. And so it turns out. We don’t at first know the impact ofthe information about the different types and numbers of animals, but your hunchmight’ve been that in some way it impacts the placement. And that hunch turns out to becorrect too. The Key Issues will be:

1) How many animals must or can be in each cage?2) Which animals must, could, or cannot be caged with each other?

Because of the unusually large number of entities (14) and the odd number breakdown ofanimal types (3/3/3/5), you might have wanted to save this game till later in the 35minutes. Game 3, and especially Game 2, might have struck you as more appealing interms of starting off the section.

The Initial Setup: You’ll likely want to picture the four cages as small boxes or circles,and label them, and make a roster of the entities. Since the cages are capitalized already,perhaps using lower-case letters for the 14 animals makes sense.

g g g h h h l l l s s s s s

W X Y Z

The Rules:

1) Rule 1 is best left till last. It’s too vague. “Two, four, or six per cage”? Let’s pass over thatuntil we can make it more concrete.

2) This rule’s two clauses make it clear that wherever you find a gerbil (g) you will find atleast one hamster (h), and vice versa. So right away, we see that g’s and h’s are linked.Consider the contrapositives here: If a cage lacks a hamster, then it cannot contain agerbil. And if it lacks a gerbil, then it cannot contain a hamster. This cements the linkeven more.

3) gives us a similar link for lizards and snakes. It works, logically speaking, exactly as Rule2 works. The presence of a lizard requires the presence of a snake, and vice versa; and ifa cage lacks one, it must lack the other.

4) Finally, some concreteness. If neither Y nor Z contains a gerbil—remember to turnnegatives into positives—then all the gerbils must go into cages W and X. You can notethat under your drawn cages W and X: “All g’s.”

Page 66: Prep Test 6

LSAT PREP _______________________________________________________________ LSAT Test VI Explained: Section IV

64 © K A P L A N

5) Same as Rule 4, different terms. Since cages W and X fail to contain lizards, all thelizards will be found in cages Y and Z. So note that in your sketch of those cages: “Alll’s.”

Key Deductions: Rule 4 links up with Rule 2, and Rule 5 with Rule 3. Let’s take them stepby step and in turn.

There are no gerbils in cages Y and Z, but without a gerbil in a cage we cannot place ahamster there. Conclusion?: There can be no hamsters in cages Y and Z either. Therefore allof the hamsters and gerbils will be placed in cages W and X.

Same deal with Rules 5 and 3. Cages W and X lack lizards, which means (Rule 3) thosecages must lack snakes as well. Hence all the lizards and snakes will occupy cages Y andZ. Now, finally, Rule 1 can help us. No cage is empty, and the minimum number ofcreatures per cage is two. Well, given the interrelationships of the hamster/gerbil andlizard/snake pairs, we can include this much as minimum information:

Wall gh

Xall gh

Yall ls

Zall ls

g+h g+h l+s l+s

What’s left? There’s one gerbil and one hamster left for cages W and X. They can’t be splitbetween those cages (that’d make for 3 creatures per cage, a violation of Rule 1). So one ofthe cages will get the remaining g+h pair. The outcome is very neat: Of cages W and X, onewill get just g+h, the other will get gghh.

Things are slightly less neat for cages Y and Z. Four creatures (one lizard and three snakes;see above sketch) remain. They will have to be placed in cages Y and Z. But how? And inwhat numbers? Each cage contains its required one snake/one lizard minimum—all that’sprohibited is that a cage contain snakes only or lizards only. So those four remainingcreatures can be placed in either cage Y or Z, as long as Rule 1’s number rules are followed.To be precise, either Y and Z will get 4 creatures apiece, or one will get 2 and the other 6.

Page 67: Prep Test 6

LSAT PREP _______________________________________________________________ LSAT Test VI Explained: Section IV

© K A P L A N 65

The Final Visualization:

Wall gh

Xall gh

Yall ls

Zall ls

g+h g+h l+s l+s

➚➚ ➚➚ g+h l s s s

2/4/6 in each!

The Big Picture:

• As you start reading the opening ¶ of each game, be ready to make a snap decision:Is this a game worth doing now? Or one worth skipping till later? Factor in thenumber of entities, the game’s familiarity, the clarity and simplicity of the game’saction, the amount of reading the game requires, and the number of questions atstake.

• Get out of the habit of working on the rules in the given order just because they’reprinted that way. Seek out the most concrete and helpful rules, and work with themfirst. They’ll give you a solid foundation to build on.

The Questions:

1. (A)As long as you consulted your sketches carefully and used the Kaplan method for thisacceptability question, you should have been OK. (E) directly violates Rule 5 by givinglizards to cage W. Check the arithmetic (Rule 1): (B) is wrong to give a cage three animals,and (C) is wrong to give a cage five. Finally, look at the distribution: If cage Y gets threelizards (D), then it gets all the lizards, and that leaves cage Z with none. Can’t be: (D) wouldmake it impossible to form a valid cage Z. (A) is correct by default.

• Just because a question looks like it’s going to be easy, don’t assume that you canafford to rush and get careless. Low-difficulty questions like this one turn intogoose-eggs when the examinee gets sloppy.

2. (D)The first if-clause is easier to deal with: If cage W gets two hamsters, then it must get twogerbils as well (to avoid violating Rule 1). That leaves a single gerbil and hamster for cageX—which, the stem goes on to tell us, has the same number of gerbils as cage Y has snakes.One! Cage Y gets a single snake, which means that all of the remaining snakes go into cageZ; five minus one is four, choice (D).

• A “two-part” question stem must be treated the same way as a “two-part” rule:namely, in two parts. Don’t lump all the information together. Take it step by step,and always begin with the information that’s more concrete.

Page 68: Prep Test 6

LSAT PREP _______________________________________________________________ LSAT Test VI Explained: Section IV

66 © K A P L A N

3. (C)The arithmetic here may be even easier than that of Q. 2. There are only three lizards tobegin with, so the if-clause here must mean that Y has one lizard and Z two. (Note that thatmakes (A) wrong automatically.) The five snakes can be divvied up among cages Y and Zin several ways, as long as each cage contains at least one snake and an overall total of two,four, or six animals. (C) works fine: The two cages could indeed have four animals each—LSSS in Y, and LLSS in Z—so (C) is the statement that “can be true.”

(B) Exactly two snakes + Y’s one lizard would result in three animals in cage Y, a violationof Rule 1.

(D) has the same problem as (B): If we add exactly three snakes to the two lizards that wedetermined must be in Z (in order to satisfy the stem), we end up with an odd number ofcritters in both Y and Z, in violation of Rule 1.

(E) Given the stem, Z has to have two lizards to Y’s one, and Z needs snakes beyond that.So (E) is impossible.

• Create a separate sketch for questions like this. Recopy the “master sketch” andinclude the question’s new data. Doing so will take much less time and avoid error.

4. (D)Looking at the master sketch, if W and Z are to get the same number of animals, thatnumber must be two or four—and if you sketch the two possibilities out briefly, it maymake the work go quicker.

If W and Z each get two, then each of them gets its minimum allotment of a gerbil andhamster for W and a lizard and snake for Z, leaving X and Y with all of the remains. If Wand Z each get four, then W gets two gerbils and two hamsters, while Z gets somecombination of lizards and snakes equaling four. Your best bet is to check each choiceagainst these possible arrangements:

(A), (B) A glance at our master sketch reveals that there are only six animals total (3g, 3h) tobe split between cages W and X. If either cage got all six, then the other would be left withzero, in violation of Rule 1.

(C) In option 1 above, Y gets four snakes, and in option 2, Y gets at least two snakes. Exactlyone snake for Y is therefore not an option.

(D) works: Y could get a lizard and three snakes, with Z getting two of each (and W gettingits four animals). On test day you need look no further, but for the record:

(E) In option 1, Z has just one snake, and in option 2, Z gets at most three snakes.

• Sometimes you have to work out two possible scenarios, but you’ll find it doesn’ttake too long if you don’t hesitate. In the time you spend trying to figure it all out inyour head, you may as well jot down the possibilities. When you’re done, it’s simplya matter of comparing the choices to your scratchwork.

Page 69: Prep Test 6

LSAT PREP _______________________________________________________________ LSAT Test VI Explained: Section IV

© K A P L A N 67

5. (E)Cage Y’s six animals would have to include the LS pair that’s always there, and the fourremaining animals, giving us LLSSSS in Y, and LS in Z. (E) is therefore correct—exactlyone of each type is present in Z. (D) is flat wrong, and so is (C): Z gets only one snake. All ofthis distribution, of course, has no impact on cages W and X. In point of fact either (A) or(B) has to be true and the other false, but we can’t tell which is which, and the Y/Z situationdoesn’t help any.

• Don’t stray from all the work you did in constructing your Final Visualization. Wedo all that work up front for a reason—it allows us to hit the ground running onquestions like this. Six animals in Y, you say? A glance at our master sketch tells usthat the breakdown has to be LLSSSS in Y and LS in Z, which leads directly to aquick point.

6. (D)As we just saw in the previous question, Y can contain four snakes and two lizards, leavingone snake and one lizard for Z. The maximum number of snakes for Y is 4, choice (D).

• The testmakers don’t always save the most difficult questions of the game for last.Often, and especially in tough games like this one, they reward those who get to thefinal questions, possibly as a result of holding off on some of the earlier ones. Here,the final two questions were eminently doable, if you kept your wits about you—which, in this case, means simply consulting your master sketch and using yourprevious work.

Page 70: Prep Test 6

LSAT PREP _______________________________________________________________ LSAT Test VI Explained: Section IV

68 © K A P L A N

Game 2—Soda Rankings(Q. 7-12)

The Action: The action here is revealed in the second sentence: The seven proposed sodanames are ranked in terms of their popularity, from first to seventh. This is straightforwardsequencing. The Key Issues will be:

1) Which names are more or less popular than which other names?2) Which names must, can and cannot be ranked in each slot from first to last?

The Initial Setup: Make a roster of the entities, but then look at the rules: Each one relatesone or more entities to others. There are no rules such as “Nipi is ranked third or sixth”—infact, only one rule directly relates to any entity’s position in the ranking, and that onenegatively (“Nipi is not 7th”). What we have here is a loose sequence, also known as a “free-floating” sequence. So we don’t want to create a vertical or horizontal list of slots just yet;rather, we want to put the rules together en route. Let’s see how this happens, working withthe most concrete rules first.

The Rules:

1) On the roster, Jazz is higher than Oboy. How much higher? No way to tell. They couldbe consecutive, or widely separated. To take note of this on the page (and to avoidthrowing yourself mixed messages), jot down the J/O relationship kind of freely,something like this:

J|O

2) Ordinarily we’d seek out another rule that ties in with what we already know. Here, thetestmakers give us a break and throw it in right away: The same Oboy whom we justlearned is outranked by Jazz, ranks higher than Kola. Therefore:

J|O|K

Page 71: Prep Test 6

LSAT PREP _______________________________________________________________ LSAT Test VI Explained: Section IV

© K A P L A N 69

3) And now Rule 3 is obliging too. Since Kola ranks higher than Mist, we can be sure of thefollowing:

J|O|K|M

—and voila, four of our seven are solidly ranked!

4) deals with Nipi, who hasn’t entered the relative ordering yet. Let’s postpone this one fornow.

5) requires care. It’s complex, and if you blow it you can write the game off. Let’s take itone step at a time—and the best place to start is with any mention of the names we’vealready heard. For that reason, let’s start with the information that Ping got more votesthan Oboy, since Oboy’s already in the picture. Jazz got more votes than Oboy as well.Who’s ranked higher, Jazz or Ping? Can’t tell yet, but we do know this:

J P \ /

O|K|M

Next: Now that Ping’s in the picture, we can incorporate the fact that Ping receivedfewer votes than Luck, but more than Nipi:

L /

J P \ / \

O N|K|M

Back to Rule 4. Nipi doesn’t come in 7th. So who does? Everyone else, except for Mist, isranked above one or more names. Therefore, it has to be true that Mist was the leastpopular name, #7 on our hit parade.

Page 72: Prep Test 6

LSAT PREP _______________________________________________________________ LSAT Test VI Explained: Section IV

70 © K A P L A N

Key Deductions: Well, we’ve already begun those, haven’t we, as we moved seamlesslyinto Step 4. (That’s how it happens with some games, notably loose sequences). In free-floating sequence games, we usually form a few deductions right off the bat simply byestablishing the relationships on the page. For example, we weren’t explicitly told, but candeduce from our sketch, that Ping received more votes than Kola, and Luck more thanNipi, Oboy and Kola. There are others, of course, and they should all be clear from therelationships etched in the sketch. It’s also valuable to consider the game’s overallparameters, for instance noticing that there are two and ONLY two contenders for the #1slot—Jazz and Luck—so I guess we’ll all be drinking Jazz or Luck sometime soon. Beyondthat, all of the other names’ rankings are limited only by what we have learned so far, whichmeans there’s still a lot of flexibility.

The Final Visualization: We should be ready to redraw our sketch as needed, when newinformation arrives. But we should also be ready to get some quick points from what we’vedone so far, which is:

L /

J P \ / \

O N|K|

7th M==

The Big Picture:

• Before you draw a sketch for a sequence game, determine whether assembling a rowor column of concrete slots makes sense. In a “loose” sequence game—one in whichthe rules relate the entities to each other—you’re better off building the relationshipsthe way we did above. (In essence, what we did was to combine Steps 3 and 4 of theKaplan Method. Ordinarily doing so is dangerous, but, as mentioned above, itusually works well for loose sequences.)

• Most games of “ranking” are best sketched out vertically.

• Take apart all multi-part rules, such as Rule 5 here, and treat them as separate piecesof information. That way you won’t run the risk of missing an important nuance.

• Turn negative rules into positives by asking the right questions. When you’re toldNipi isn’t ranked 7th, two questions should immediately come to mind: “Who isranked 7th?” And “Where is Nipi ranked?” By asking them and following them up,you can make Big Deductions . . . and rack up big points.

Page 73: Prep Test 6

LSAT PREP _______________________________________________________________ LSAT Test VI Explained: Section IV

© K A P L A N 71

The Questions:

7. (C)Ordinarily, in an Acceptability question, you should check each rule against the choices toseek the violators. Here it’s even easier, since we know we must see J--O--K--M in thatrough order somewhere among the right answer, and (A) and (D) both violate thatsequence, so they can go in one swoop. (B) violates Rule 4 (not to mention the fact that wededuced that Mist must be ranked 7th). And (E) reverses Rule 5’s requirement that Ping getfewer votes than Luck. As the only one left standing, (C) must be right.

• Move swiftly in Acceptability questions to apply the Kaplan technique ofcomparing what you know to the choices, and throwing out the violators.

8. (E)(E)’s relationship is easily seen in the sketch or deducible like so: P gets more votes than O(Rule 5) and O gets more than K (Rule 2), hence . . . choice (E).

(A), (B), (D) We cannot be sure of Nipi’s precise relationship to Jazz, Oboy, or Kola. Thesethree statements are possibly true, but not necessarily so.

(C) would be true if Luck is ranked #1. But if Jazz is ranked #1, then (C) is false.

• Don’t jump into drawing a new sketch until you’re sure that you need one. Manyquestions, like this one, are manageable by just consulting your master sketch.

9. (C)You can sketch out the question stem like so:

POK

Next you can add the things you already know—and remember, the stem mandates that “PO K” is an unbroken sequence:

J L \ /

POK|M

Page 74: Prep Test 6

LSAT PREP _______________________________________________________________ LSAT Test VI Explained: Section IV

72 © K A P L A N

All that’s left is to add N, which ordinarily will rank somewhere below P (Rule 5) but not7th (Rule 4). But here, since “P O K” is an unbroken sequence, we can be sure of:

J L \ /

POKNM

(A) is possibly true (if Jazz is ranked #1), while (B), (D), and (E) are all definitely true in thissequence. But (C) is false: Oboy must rank above Nipi here.

• When a stem gives you concrete information—as this one does—jot down its gist.Then proceed to the overall rules and put it all together.

10. (A)We saw this up front: All we can be sure of is that Mist got the fewest votes. Perhaps youran with this and chose (A) and immediately moved on to save time. Kudos if you did! Ifnot, you probably double-checked to confirm that every other entity has at least twopossible rankings. N’s ranking is unfixed relative to that of J, O, and K, so the exact positionof these four names can’t be determined without more info. L can be first or second, whileP could be second or third. Only M’s dead last ranking can be precisely determined fromthe partial results in the rules—choice (A).

• An excellent example of how working out possibilities during your setup work canlead to quick and easy points during the game itself.

11. (B)We’ve already seen, in Qs. 7 and 9, that Luck, Ping, and Jazz can all rank among the topthree, and we know from the setup that Mist must rank 7th. So all we need do here is figureout which of the remaining names—Kola, Nipi, and Oboy—could sneak into the top three.Kola cannot, because at least four other names (J, O, P, and L) rank higher. Oboy cannot,because at least three others rank higher (J, L, and P). But Nipi can, since it is only for-sureoutranked by Luck and Ping; Nipi could rank 3rd. Final box score: Three names (M, K, O)cannot be among the top three; four can—choice (B).

• In non-if questions, there’s no new material to incorporate into the arrangement.Instead, use your master sketch and previous work to narrow down the number ofchoices and/or entities that you need to test for the particular circumstance askedabout in the stem (here, the names that could rank in the top three).

Page 75: Prep Test 6

LSAT PREP _______________________________________________________________ LSAT Test VI Explained: Section IV

© K A P L A N 73

12. (B)Create a new sketch here, putting P atop J. That means that L, always atop P, is now at thetop of the list. It reads: L--P--J--O--K--M. But what about Nipi? As long as its rank is belowPing’s (Rule 5) and not last (Rule 4), it can take any position. So all we know for sure is thatL is 1st, P 2nd, and M (as always) 7th. That’s three, choice (B). Positions 3 through 6 remainuncertain, because N can be inserted anywhere in this middle part of the ordering.

• Use your pencil! The extra time you take to keep track of the possibilities will berewarded. Sketch out the new relationships when the stem posits a new condition.The perils of trying to manipulate the master sketch in your head are legion: apossible waste of time; a likely loss of a point.

Page 76: Prep Test 6

LSAT PREP _______________________________________________________________ LSAT Test VI Explained: Section IV

74 © K A P L A N

Game 3—Park Benches(Q. 13-19)

The Action: Nothing could be clearer than the simple opening sentence andaccompanying sketch. We have to place the eight benches around the park, one for each ofthose large black dots. Peeking ahead at the first indented rule tells us that it’s a little morecomplicated than one bench/one spot—we have to consider the colors of the benches aswell—but still and all, the overall task should be blazingly clear, as are the Key Issues:

1) Which benches must, cannot, or can be placed in a certain location?2) What is the relationship of each bench color to the other colors in terms of placement?

The Initial Setup: Indented Rule 1 comes into play here, of course. You want to make aroster of the entities, and perhaps you found a way to distinguish among the differentcolors. Often, when there are two types of entities we can get by with using caps and lower-case, but the problem of representing the third type here may have made the whole thingtoo complicated. For the sake of this explanation, we’ll simply keep them all in caps:

GREEN: J K LRED: T UPINK: X Y Z

The rules, of course, will determine how we execute the task: placing the 8 benches into the8 slots. Many students choose to move to the most concrete rules first, and so shall we:

The Rules:

5) places T in the lower right corner. Draw T there, and cross T off the master roster.

6) In the same way, we can place J where the rule indicates, right at the printed word“North.” Cross J off the roster too.

2) tells us that J, K, and L all stand adjacent to each other. Now that we have seen that Jstands in the middle of the north edge, we know that K and L will stand nearby. Butwhere? Too early to tell. Keep in mind, though, the JKL relationship.

3) is similar to Rule 2: The pink benches, namely X, Y, and Z, also have to be placed in threeadjacent slots; though, again, we cannot know in what order.

4) is key. The green and pink benches can’t stand next to each other. So our 3 greens in arow cannot stand next to our 3 pinks in a row. What’s going to separate them? Has to be thered ones, no? In other words, we’re going to have to have a red bench, the three greens, thenthe other red, then the three pinks. There’s no other way. Check it out.

Check out, too, how sometimes one is moved to proceed into Step 4 of the Kaplan Methodbefore Step 3 (the work with the individual rules) is done.

Page 77: Prep Test 6

LSAT PREP _______________________________________________________________ LSAT Test VI Explained: Section IV

© K A P L A N 75

7) doesn’t help us very much, being highly conditional. If T and X are adjacent, then wecannot have T adjacent to L. Beyond that, we ought to think through and jot down thecontrapositive (If we ever see TL, then we cannot have TX) and leave it go until it has to bebrought into play. For now, there’s more to be done in Step 4.

Key Deductions: We’ve seen that the red benches will be needed to separate ourconsecutive green and pink benches. Now we need to recall that one of those two redbenches, T, stands in the southeast corner. It follows that to the left of ‘t’ will be one of ourconsecutive trios, either the greens or pinks, and to the right, the other consecutive trio.This leaves the northwest corner open, and one and only one red bench to fill it. It’s a hugededuction that bench U must stand in the northwest corner, and hopefully this was thededuction you made and incorporated right into the picture on the page.

At the same time, J stands up there too. So we know (relatively) where the greens go, andhence where the pinks go. Be as definite as you can:

U J K/L

X/Y/Z K/L

X/Y/Z X/Y/Z T

And there you have it: The Final Visualization. Rule 7 now makes a bit more sense: If Tstands next to X, the rule says, then L doesn’t stand next to T...which will mean that K doesstand next to T, since that’s the only other contender. But again, that’s much more “what if”speculative work than we need to engage in. When Rule 7 comes into play, we’ll know it.Let’s first get some “gimmes.”

The Big Picture:

• Typically, the testmakers give us a picture when describing the situation in prosewould be too wordy and confusing. Take advantage of any picture that thetestmakers decide to provide: It does a lot of the work for you.

• Turn all negative rules into positives. Based on what CANNOT be true, figure outwhat CAN or MUST be true.

• Be guided every step of the way by the game’s action. Ask yourself with every rule atevery moment: How can this help me execute the game’s action? Here, we want to askourselves constantly, “How can this help me place the benches?”

Page 78: Prep Test 6

LSAT PREP _______________________________________________________________ LSAT Test VI Explained: Section IV

76 © K A P L A N

The Questions:

13. (E)Our sketch makes clear that one of the two green benches K or L has to stand in thenortheast corner. They chose the latter for correct choice (E).

(A), (B), (C) As we deduced above and represented in the Final Visualization, these threepink benches have to occupy the bottom left corner of the square.

(D) Rule 5 requires us to place T at the southeast corner.

• “Gimmes” are ever present in Logic Games. Find ‘em and get ‘em!

14. (A)Since the four wrong choices are positively-true statements, it follows that the correctanswer is either downright false, or only possibly true. We get the former in correct choice(A): As we deduced above, the bench in the northwest corner is red bench U.

(B), (D) Yes, each of these benches is green: K and L in either order.

(C), (E) Yes, these two benches, along with the middle bench on the south side, are all pinkbenches X, Y, and Z in some order.

• Take the time throughout your Logic Games work to characterize each question’sanswer choices (“What is true of the right answer?” “What must be therefore true ofthe wrong answers?”). This is time well spent.

15. (D)Ka-ching! The sketch instantly reveals our Big Deduction that U, in the northwest corner,stands next to J. Either K (A) or L (B) must stand on the other side of J in the northeastcorner, but we cannot tell which. And T (C) and X (E) are safely separated from J.

• A question like this—asking for something that MUST be true without providingany “if” information—is a good clue (if you need one) that there is at least one BigDeduction to be made during Step 4 of the Kaplan Method.

16. (A)Reading comprehension is the key here. We are asked for a bench that has exactly twopossible locations: not one, as T (B) has, or three, as X, Y, and Z have (C), (D), and (E). Aswe’ve seen in the setup and again in Q. 13, K and L will occupy the two spots between Jand T on the east side of the park, but we can’t be sure which stands where. And as with Q.13, either K or L could’ve been chosen as the answer; this time they happen to have chosenK for correct choice (A).

Page 79: Prep Test 6

LSAT PREP _______________________________________________________________ LSAT Test VI Explained: Section IV

© K A P L A N 77

• When a stem is complex or lengthy, the worst thing you can do is take it for granted,rushing through it or reading it carelessly. Time spent thinking will pay off in ahigher score.

• Critical reading skills are essential not only in Logical Reasoning and ReadingComprehension, but here in Logic Games as well. Boil complicated phrases down assimply as possible: “. . . two and no more than two . . .” MEANS exactly two.

17. (D)Before you draw anything, take a look at the master sketch with this question’s if-clause inmind. We see where the three pink benches are: the southwest corner, and the spots oneither side of it. How, then, can pink Z stand directly north of pink Y? Only one way: if Ystands at the southwest corner, and Z stands right above it. This leaves X for the middle ofthe south row, next to T...and if we are keeping an eye on the choices, we see that (D) istrue. As the bench in the middle of the south row, X is in fact due south of J, the bench inthe middle of the north row.

If you didn’t stop to check the choices, there’s more work that can be done. Since T herestands next to X, L cannot; which in turn means that K must stand next to T, leaving L forthe northeast corner. Many of the wrong choices hinge on all of that:

(A) No, J is northwest of K.

(B) No, K is southeast of U.

(C) No, U is northwest of X.

(E) No, Z is southwest of J.

• Before you draw anything, look at your master sketch with each question’s new stemin mind. That thinking will guide your sketch, saving you time and avoidingmissteps.

18. (B)Immediately draw your eight dots next to the question, enter Y where you’re told to, andrecopy what you already know:

U J K/L

Y K/L

X/Z X/Z T

Page 80: Prep Test 6

LSAT PREP _______________________________________________________________ LSAT Test VI Explained: Section IV

78 © K A P L A N

We’re looking for a pair of benches that CANNOT stand in the corners, and notice that theonly rule still up in the air is the last one. The big insight here is realizing that one of thechoices must force a violation of Rule 7. And (B) certainly would: If Z and K are in thesouthwest and northeast corners, respectively, then we are left with X and L standing nextto T—which is specifically prohibited by Rule 7.

• Better to take a few extra seconds to recopy a new sketch for a question like this, thanto do all the work in your head and risk wasting time and picking a wrong choice.

• Good test-takers develop a sense of which rules are relevant when. Here, the key wasto notice that once the entities were placed as in the sketch above, only Rule 7remained.

19. (C)Check your master sketch for the possibilities for Y, L, and T. The only way for Y tosimultaneously stand farther south than L and farther north than T is if their position is:

. . L

Y .

. . T

You can enter in the other data we know for sure: U in the northwest corner; J standingbetween U and L; now we know that K has to stand between L and T on the eastern side.The only uncertainty is between X and Z; there’s no danger of violating Rule 7 now that Tand L are separated, so X and Z can take the two remaining spots in either order. As aresult (C) is correct, since X CAN stand next to T but need not. All of the other choices areadjacent pairs. (Note, for (E), that no matter in which order we place X and Z, they’re stilladjacent.)

• You can start a new question’s sketch—as we did here—by working out, on paper, itsspecifics. Then build in what you already know.

Page 81: Prep Test 6

LSAT PREP _______________________________________________________________ LSAT Test VI Explained: Section IV

© K A P L A N 79

Game 4—Island Bridges(Q. 20-24)

The Action: As the game begins, five islands are unconnected by bridges, but contractorsare about to remedy that. In a way we are acting as surrogates for the bridge planners: Weneed to connect the bridges appropriately. Though this is a game situation you probablyhave not seen before, the Key Issues would seem to be pretty clear, namely:

1) Which islands must, can, or cannot be connected by bridge to which other islands?2) How many bridges must, can, or cannot an island have?

The Initial Setup: When a game mentions, or seems to hinge on, a map, you might as welldraw one. Since we’re not told anything about the islands’ spatial relationship to eachother, and that issue never comes up, it doesn’t matter exactly how you locate them; in thissituation, most people tend to try setting the entities in a rough circle. Where to start? Asneak peek at the concrete Rule 7 might suggest that you locate J and M close to O. We canalso throw K and L in to complete the circle, and move them elsewhere later on ifnecessary:

J \ K O

/ L M

(If you didn’t see this, or if some new information comes along, you can always redraw thearrangement—doing so won’t take long.) Anyway, connecting the letters by straight lineswould seem adequate to indicate the bridges.

The Rules:

We’ve already taken care of Rule 7, and so can now fill in the other information around thetentative sketch above:

1) This rule is a loophole-closer—meaning a rule designed to define the setup and limitpossibilities that common sense would otherwise leave open. Because of Rule 1, weknow that every bridge connects just 2 islands (unlike, say, New York’s TriboroughBridge, which as its name indicates connects three bodies of land), and that no bridgesintersect.

2) Another loophole closer. Students have always reported having trouble with this one.But the testmakers are trying to make things clear; maybe the best way to understand itis to try a concrete example or two. Suppose a bridge is built between M and O. Well,then, all the rule is there for is to prohibit any other M/O bridge: One bridge per pair ofislands. That’s it! It means that any bridge that connects islands is a single bridge. Again,just a loophole closer meant to forestall any ambiguity in the numbers that govern thegame.

Page 82: Prep Test 6

LSAT PREP _______________________________________________________________ LSAT Test VI Explained: Section IV

80 © K A P L A N

3) Not a loophole closer this time, but a concrete piece of information that guides theaction: The maximum number of bridges that can be built to any one island is three. Soeach island will be connected to three islands maximum. Make a note of it.

4) is vague enough that we might profitably skip it for the moment, and move to moreconcrete rules.

5) “J has exactly two connections.” Well, one of them is established by Rule 7. So we needto link up J to one more island. Which one? No way to tell. Let’s just note it off to theside:

(+ 1) J \ K O

/ L M

6) “K gets just one bridge.” Fair enough. We don’t know which one it is, so an annotationwill do for now:

(+ 1) J \

(1 only) K O /

L M

4) Now back to 4: Of J, K, and L, each one has to link up to M or O or both. A THREE-partrule: Three rules in one. Well, we have a context for that fact now—namely, all the otherthings we’ve learned—so let’s explore this rule in Step 4 in the context of all the otherinformation:

Key Deductions: Starting with J in Rule 4: “J must be connected to M or O or both.” Well,that minimum requirement has been met (Rule 7): J is connected to O. So that’s all we cando with J and this rule. J’s other bridge could link J to M, but needn’t.

“K is connected to M or O or both.” Well, K only gets one bridge, so Rule 4 tells us it has tobe a K/M or K/O bridge. So our sketch can be adjusted to reflect that:

(+ 1) J \

(1 only, to M or O) K O /

L M

(And you may wish to note that K/J and K/L bridges are therefore prohibited.)

Page 83: Prep Test 6

LSAT PREP _______________________________________________________________ LSAT Test VI Explained: Section IV

© K A P L A N 81

Continuing with Rule 4: “L is connected with M or O or both.” L is otherwise wide open; Lcould have a minimum of one bridge (L/M or L/O), or two, or the maximum of three.

Anything else? We should notice that O, with 2 bridges already, can only have one moremaximum; and that M, with 1 bridge already, can get zero, one, or two more.

We should also notice that indeed every island has to have at least one bridge. That wasn’tspelled out as a rule, but proves to be true.

The Final Visualization:

(+ 1) J \ 3 MAXIMUM!!!

(1 only, to M or O) K O /

L M (M or O or both)

The Big Picture:

• There are always four basic questions that you have to ask and answer as a gamebegins: What’s the real-life situation? What are the entities? What’s the action? Andwhat is the arithmetic—the restrictions on the action? These questions are especiallyimportant when, as here, the game and/or the situation are unfamiliar. The answersto these questions will take the game from the strange to the manageable.

• Another key to getting an unfamiliar game under control is the making of mentalpictures. See those islands and bridges in your mind’s eye. Make it make sense. Andremember: The game must work; it wouldn’t be on the LSAT if it didn’t.

• Feel free to peek ahead at the rules in your work on Step 2 of the Kaplan Method. Ifin this game you look ahead, for instance, and see that there’s nothing said about thephysical relationship of the islands (e.g. which is north or south, number of milesbetween, etc.), you can more confidently construct a sketch.

• Give yourself the freedom to attack the rules in the most helpful order. Establishingconcrete relationships in the early going will help you to handle the more abstractrules by providing a foundation on which to build.

Page 84: Prep Test 6

LSAT PREP _______________________________________________________________ LSAT Test VI Explained: Section IV

82 © K A P L A N

The Questions:

20. (D)There will be no K/L connection, since K gets only one bridge connection (Rule 6) and thatmust be to M or O (Rule 4). But nothing stops L from being connected to J, O, and/or M.Any or all of them can link up with L—choice (D).

• Every game has one or more “gimme” questions, and sometimes (as here) they startoff the set of questions. Be ready to capitalize on this fact.

21. (C)Since the right answer CAN be true, it follows that the four wrong choices are impossible.This is a variation of the standard “acceptability” question. You can check each rule againstthe choices, or simply test each choice against the master sketch above. Either way, youshould see rather readily that (C) is acceptable (L can have J and M connections with nodamage to the rules) and that:

(A) Since J is already connected with O, connecting J with L and M would be a total ofthree bridges, violating Rule 5.

(B) Since K only gets one connection (Rule 6), (B) is a blatant violation.

(D) Since M is already connected with O, adding (D)’s connections would give M a total offour bridges, violating Rule 3.

(E) Since O is already connected with J, (E) would give O a total of four bridges. Again, aviolation of Rule 3.

• Some students prefer to postpone a question this vague until later in the game,figuring that it’ll be easier once they learn more about how the game works. Not abad strategy. But sometimes a “could be true” question is much like an acceptabilityquestion in disguise.

22. (B)Creating a new sketch for this question is advisable. In it, repeat the connections we’re sureof (J/O and M/O), and add the new K/O bridge. Two things should now be immediatelyapparent: (1) That’s it for K, which has the one connection permitted it by Rule 6. (2) That’salso it for O, which has its total of three connecting bridges permitted by Rule 3. L’s theonly island not yet hooked up—and all that’s left are J and M. L will have to link up M(Rule 4), and could link up with J as well. Now look through the choices for a possiblestatement.

(A) Impossible. Not establishing an L/M bridge, with O unavailable to hook up with L,violates Rule 4.

(B) Possible, as noted above. There’s our answer.

Page 85: Prep Test 6

LSAT PREP _______________________________________________________________ LSAT Test VI Explained: Section IV

© K A P L A N 83

(C) Impossible. O’s quota is reached with connections to J, K, and M.

(D) Impossible. L has a maximum of two connections (J and M) here.

(E) Impossible. O, as we’ve seen, has three bridge connections—two from Rule 7, and onefrom the question stem.

• Every word in a Logic Games stem can have import. If you miss the simple word“could” here, you can waste lots of time and even get the question wrong. Don’t rushyour analysis of the stems.

23. (B)The stem is very concrete. Create a new sketch with the new and old informationrepresented. You see that O has reached its limit (J, L, M = three bridges) and that K is stillunconnected by any bridges. That won’t do; and since Rule 4 dictates that K be connectedwith either O or M, and O is once again maxed out, there’s no way around it—K must linkup with M, choice (B).

(A) and (E) are both impossible: Since M must link with K here, M now has threeconnections—K (see above), L (from the stem), and O (Rule 7). So (E) is dead wrong—M hasthree, not two connections; and (A) is impossible, since M is now maxed out and thereforecannot connect with J.

(C) is impossible, since O’s limit of three bridges has already been reached through J, L,and M. As described above, this is the fact that forces the KM connection, and hence correctchoice (B).

(D) turns out to be impossible as well: If we follow this scenario through to the bitter end,we see that J’s required second connection (Rule 5) must be with L, what with K, O, and Meach connected to the max. So J must connect to L, which means that L now has threeconnections itself (including the two in the stem), not two as (D) would have it.

• When you spot a deduction, scan for it among the choices. If it’s there, circle it andmove on to save time. Your job in Logic Games is not to fully disprove all the wrongchoices once you’ve found an answer that works. Remember: Correct answers in LGare objectively correct; when you’ve found one, have the confidence to blow off therest.

Page 86: Prep Test 6

LSAT PREP _______________________________________________________________ LSAT Test VI Explained: Section IV

84 © K A P L A N

24. (A)This stem is more of a critical reading challenge than anything else. We are forbidden, inessence, to connect any island with both M and O. Anything jump out at you at this point?Hopefully, you said to yourself “hey, J is already connected to O, so we can’t have JM.”Following this line of thought, we see that since J can never connect to K—K gets only onebridge and that one has to be with M or O—and J needs two connections to satisfy Rule 5, Jmust connect to L. Voila, choice (A).

(B) Impossible. O and J are already connected by a bridge; (B) would violate the questionstem.

(C) is possible but not necessarily true, since K’s one bridge could be built to M just asreadily as to O.

(D), (E) L needs a bridge connection with M or O (Rule 4) and cannot be connected withboth (according to the stem). But which bridge is built? We cannot be sure.

• A hallmark of high-difficulty Logic Games questions is that they involve many ofthe rules simultaneously. Make it a goal to become adept at using the clues in thestem to lead you to the most relevant rules first. Part of Logic Games success ishaving a good sense of what rules are put into play at what point. If you develop agood feel for this, you won’t find yourself hesitating as much about where to start, orwhere to go next.

Page 87: Prep Test 6

I.N. LL3089 Rev.B Printed in the USA